[obm-l] Re: [obm-l] Fwd: Módulo

2022-08-11 Por tôpico Carlos Gomes
Ola amigo. Normalmente essas equações diofantinas nao lineares tem solução
passando por congruência.



Em qui., 11 de ago. de 2022 16:11, Esaú Gomes 
escreveu:

> Alguém poderia me falar o que estudar mais especificamente na questão
> abaixo?
>
> Para quais valores naturais  de *n* e *x*, existe solução
> 2^n = 3x + 1.
>
> --
> Esta mensagem foi verificada pelo sistema de antivírus e
> acredita-se estar livre de perigo.

-- 
Esta mensagem foi verificada pelo sistema de antiv�rus e
 acredita-se estar livre de perigo.



Re: [obm-l] Trascendencia

2021-03-29 Por tôpico Carlos Gomes
Rapaz o melhor lugar em Portugues é a RPM online ou a Matemática
universitária. Em inglês, mas bem concorrida é a American Mathematical
Monthly.

https://pmo.sbm.org.br/
https://rmu.sbm.org.br/
https://www.tandfonline.com/toc/uamm20/current

Em seg., 29 de mar. de 2021 às 16:11, Israel Meireles Chrisostomo <
israelmchrisost...@gmail.com> escreveu:

>
> Acho que consigo provar a transcendência de pi, como faço para publicá-la?
> --
> Israel Meireles Chrisostomo
>
> --
> Esta mensagem foi verificada pelo sistema de antivírus e
> acredita-se estar livre de perigo.

-- 
Esta mensagem foi verificada pelo sistema de antiv�rus e
 acredita-se estar livre de perigo.



Re: [obm-l] Revista obm

2019-11-08 Por tôpico Carlos Gomes
Olá amigos. No mês passado foi publicada mais uma Eureka. A publicação foi
reativada.

Att, Cgomes

Em sex, 8 de nov de 2019 06:31, Anderson Torres <
torres.anderson...@gmail.com> escreveu:

>
>
> Em qui., 10 de out. de 2019 às 11:02, samuel barbosa 
> escreveu:
>
>> Já existem dois números da Eureka prontos e estão em fase de revisão. O
>> número 41 certamente será publicado em sua versão online antes da prova da
>> OBM em novembro.
>>
>> A Eureka está voltando após um hiato de quase três anos.
>>
> Previsão de retorno? Eu ainda tô a fim de escrever um artigo para eles!
>
>
>>
>> Abraços
>> Samuel
>>
>> Em sáb, 28 de set de 2019 às 14:09, Israel Meireles Chrisostomo <
>> israelmchrisost...@gmail.com> escreveu:
>>
>>> Alguém sabe se as publicações da revista OBM estão  suspensas?
>>>
>>> --
>>> Israel Meireles Chrisostomo
>>>
>>>
>>> 
>>>  Livre
>>> de vírus. www.avg.com
>>> .
>>>
>>> <#m_-5757417543590061019_m_832875430001420_m_1752696122834832852_DAB4FAD8-2DD7-40BB-A1B8-4E2AA1F9FDF2>
>>>
>>> --
>>> Esta mensagem foi verificada pelo sistema de antivírus e
>>> acredita-se estar livre de perigo.
>>
>>
>> --
>> Esta mensagem foi verificada pelo sistema de antivírus e
>> acredita-se estar livre de perigo.
>
>
> --
> Esta mensagem foi verificada pelo sistema de antivírus e
> acredita-se estar livre de perigo.

-- 
Esta mensagem foi verificada pelo sistema de antiv�rus e
 acredita-se estar livre de perigo.



Re: [obm-l] livros aparentemente interessantes

2019-04-01 Por tôpico Carlos Gomes
Olá Maurício...são todos excelentes. Tenho boa parte deles. Vale a pena o
investimento.

Abraço, Cgomes.

Em seg, 1 de abr de 2019 às 20:05, Mauricio de Araujo <
mauricio.de.ara...@gmail.com> escreveu:

> Boa noite!
>
> Alguém conhece algum dos livros presentes no link a seguir?
>
> https://www.awesomemath.org/shop/
>
> Parecem muito bons...
>
> Att.
>
> --
> Esta mensagem foi verificada pelo sistema de antivírus e
> acredita-se estar livre de perigo.

-- 
Esta mensagem foi verificada pelo sistema de antiv�rus e
 acredita-se estar livre de perigo.



Re: [obm-l] soma de tan^2

2017-09-16 Por tôpico Carlos Gomes
Olá Luis...lembro desse problema ...ele foi publicado na Mathematical
excalibur ha alguns anos https://www.math.ust.hk/excalibur/

A resposta é C(90,2)= 4005, se não me falha a memória...usa relações de
Girard num "polinômio esperto"...vou tenter ver se lembro a solução...se
lembrar ponha aqui!

Abraço, Cgomes.

Em 16 de setembro de 2017 10:48, Luís Lopes 
escreveu:

> Sauda,c~oes,
>
>
> Bom dia.
>
>
> Me mandaram a seguinte questão:
>
>
> (1) Seja S = tan²(1º) + tan²(3º) + tan²(5º) + ... + tan²(89º), calcule o
> valor de S.
>
> Como resolver ? Obrigado.
>
>
> Abs,
>
> Luís
>
>
>
> --
> Esta mensagem foi verificada pelo sistema de antivírus e
> acredita-se estar livre de perigo.
>

-- 
Esta mensagem foi verificada pelo sistema de antiv�rus e
 acredita-se estar livre de perigo.



Re: [obm-l] a quem possa interessar

2017-08-10 Por tôpico Carlos Gomes
Obrigado por compartilhar Murício...uma mina de ouro!

abraço, Cgomes.

Em 9 de agosto de 2017 22:36, Mauricio de Araujo <
mauricio.de.ara...@gmail.com> escreveu:

> https://drive.google.com/drive/folders/0B8qeUE5SqcPAWFVaM1N5anN3S2M
>
>
>
> --
> Esta mensagem foi verificada pelo sistema de antivírus e
> acredita-se estar livre de perigo.

-- 
Esta mensagem foi verificada pelo sistema de antiv�rus e
 acredita-se estar livre de perigo.



[obm-l] Re: [obm-l] Re: [obm-l] Re: [obm-l] Números triangulares

2017-08-09 Por tôpico Carlos Gomes
Não é uma pegadinha...são dois problemas completamente diferentes! O
resultado deve ser verdadeiro para números triangulares não consecutivos,
mas NECESSARIAMENTE a condição de serem não consecutivos precisa ser
explicita no enunciado, caso contrário a solução é a do Israel. Mas é
interessante no caso não consecutivo...vamos tentar...

Em 9 de agosto de 2017 22:48, Bruno Visnadi 
escreveu:

> Ainda assim, todo número natural ímpar é a diferença de dois números
> triangulares não consecutivos. O problema é uma 'pegadinha', mesmo!
>
> Em 9 de agosto de 2017 22:40, Israel Meireles Chrisostomo <
> israelmchrisost...@gmail.com> escreveu:
>
>> Se ele tivesse dito triangulares não consecutivos, aí talvez o problema
>> ficaria mais interessante.
>>
>> Em 9 de agosto de 2017 22:32, Israel Meireles Chrisostomo <
>> israelmchrisost...@gmail.com> escreveu:
>>
>>> Esse problema foi formulado de modo a enganar o leitor, ao se colocar
>>> muitos detalhes que nos confundem.Talvez o autor do problema tenha
>>> encontrado uma relação mais complexa, mas como o problema está muito
>>> abrangente, o problema se resolve facilmente por essa observação.
>>>
>>> Em 9 de agosto de 2017 22:19, Israel Meireles Chrisostomo <
>>> israelmchrisost...@gmail.com> escreveu:
>>>
 A diferença t(n+1)-t(n)=(n+1)(n+2)/2-n(n+1)/2=n+1  qualquer número
 natural maior do que 0 é a diferença de dois números triangulares

 Em 9 de agosto de 2017 21:23, Pedro Chaves 
 escreveu:

> Caros Colegas,
> Seja N = {1, 2, 3, 4, 5, ...} o conjunto dos números naturais.
> Chamamos de número triangular a qualquer número obtido pela expressão
>  t(n) = n.(n+1) / 2, sendo n um natural qualquer.
> Como podemos provar que o quadrado de qualquer número natural ímpar,
> múltiplo de 3, é a diferença entre dois números triangulares?
> Abraços do Pedro Chaves.
> 
> ---
>
>
>
>
> --
> Esta mensagem foi verificada pelo sistema de antivírus e
> acredita-se estar livre de perigo.
>



 --
 Israel Meireles Chrisostomo

>>>
>>>
>>>
>>> --
>>> Israel Meireles Chrisostomo
>>>
>>
>>
>>
>> --
>> Israel Meireles Chrisostomo
>>
>> --
>> Esta mensagem foi verificada pelo sistema de antivírus e
>> acredita-se estar livre de perigo.
>
>
>
> --
> Esta mensagem foi verificada pelo sistema de antivírus e
> acredita-se estar livre de perigo.
>

-- 
Esta mensagem foi verificada pelo sistema de antiv�rus e
 acredita-se estar livre de perigo.



[obm-l] Re: [obm-l] Re: [obm-l] Números triangulares

2017-08-09 Por tôpico Carlos Gomes
Isso mesmo Israel...eu estava exatamente tentando isso aqui!

Em 9 de agosto de 2017 22:40, Israel Meireles Chrisostomo <
israelmchrisost...@gmail.com> escreveu:

> Se ele tivesse dito triangulares não consecutivos, aí talvez o problema
> ficaria mais interessante.
>
> Em 9 de agosto de 2017 22:32, Israel Meireles Chrisostomo <
> israelmchrisost...@gmail.com> escreveu:
>
>> Esse problema foi formulado de modo a enganar o leitor, ao se colocar
>> muitos detalhes que nos confundem.Talvez o autor do problema tenha
>> encontrado uma relação mais complexa, mas como o problema está muito
>> abrangente, o problema se resolve facilmente por essa observação.
>>
>> Em 9 de agosto de 2017 22:19, Israel Meireles Chrisostomo <
>> israelmchrisost...@gmail.com> escreveu:
>>
>>> A diferença t(n+1)-t(n)=(n+1)(n+2)/2-n(n+1)/2=n+1  qualquer número
>>> natural maior do que 0 é a diferença de dois números triangulares
>>>
>>> Em 9 de agosto de 2017 21:23, Pedro Chaves 
>>> escreveu:
>>>
 Caros Colegas,
 Seja N = {1, 2, 3, 4, 5, ...} o conjunto dos números naturais.
 Chamamos de número triangular a qualquer número obtido pela expressão
  t(n) = n.(n+1) / 2, sendo n um natural qualquer.
 Como podemos provar que o quadrado de qualquer número natural ímpar,
 múltiplo de 3, é a diferença entre dois números triangulares?
 Abraços do Pedro Chaves.
 
 ---




 --
 Esta mensagem foi verificada pelo sistema de antivírus e
 acredita-se estar livre de perigo.

>>>
>>>
>>>
>>> --
>>> Israel Meireles Chrisostomo
>>>
>>
>>
>>
>> --
>> Israel Meireles Chrisostomo
>>
>
>
>
> --
> Israel Meireles Chrisostomo
>
> --
> Esta mensagem foi verificada pelo sistema de antivírus e
> acredita-se estar livre de perigo.
>

-- 
Esta mensagem foi verificada pelo sistema de antiv�rus e
 acredita-se estar livre de perigo.



[obm-l] Re: [obm-l] Re: [obm-l] Números triangulares

2017-08-09 Por tôpico Carlos Gomes
Ótima solução Israel...

Em 9 de agosto de 2017 22:19, Israel Meireles Chrisostomo <
israelmchrisost...@gmail.com> escreveu:

> A diferença t(n+1)-t(n)=(n+1)(n+2)/2-n(n+1)/2=n+1  qualquer número
> natural maior do que 0 é a diferença de dois números triangulares
>
> Em 9 de agosto de 2017 21:23, Pedro Chaves 
> escreveu:
>
>> Caros Colegas,
>> Seja N = {1, 2, 3, 4, 5, ...} o conjunto dos números naturais.
>> Chamamos de número triangular a qualquer número obtido pela expressão
>>  t(n) = n.(n+1) / 2, sendo n um natural qualquer.
>> Como podemos provar que o quadrado de qualquer número natural ímpar,
>> múltiplo de 3, é a diferença entre dois números triangulares?
>> Abraços do Pedro Chaves.
>> 
>> ---
>>
>>
>>
>>
>> --
>> Esta mensagem foi verificada pelo sistema de antivírus e
>> acredita-se estar livre de perigo.
>>
>
>
>
> --
> Israel Meireles Chrisostomo
>
> --
> Esta mensagem foi verificada pelo sistema de antivírus e
> acredita-se estar livre de perigo.
>

-- 
Esta mensagem foi verificada pelo sistema de antiv�rus e
 acredita-se estar livre de perigo.



[obm-l] Re: [obm-l] Re: [obm-l] Re: [obm-l] Re: [obm-l] Polinômios

2017-07-25 Por tôpico Carlos Gomes
Pelo teorema do resto,

p(2)=p(3)=p(4)=r e p(1)=0

Considerando o polinômio q(x)=p(x)-r, segue que q(2)=q(3)=q(4)=0. Assim,

q(x)=A.(x-2)(x-3)(x-4), com A real. Portanto,

p(x)-r=q(x)=A.(x-2)(x-3)(x-4) ==> p(x)=A.(x-2)(x-3)(x-4)+r.

Ora, como p(1)=0, segue que 0=A(1-2)(1-3)(1-4)+r ==> r=6A

Assim, p(x)=A.(x-2)(x-3)(x-4)+¨6A

Variando o A nos reais (A não nulo) temos infinitos polinômios p cumprindo
as condições requeridas.

Cgomes.

-- 
Esta mensagem foi verificada pelo sistema de antiv�rus e
 acredita-se estar livre de perigo.



[obm-l] Re: [obm-l] Re: [obm-l] Re: [obm-l] Sugestão de material para OBM

2017-07-04 Por tôpico Carlos Gomes
Concordo plenamente!

Em 4 de jul de 2017 16:02, "Pedro José" <petroc...@gmail.com> escreveu:

Boa tarde!

Desculpe-me, pela intromissão. Mas você, que não é da área de exatas, fica
difícil enveredar de cara no nível universitário.
Procure começar pelo nível médio. Os problemas já são cascas-grossas.
Primeiro se erguer, depois andar e por fim correr, é o que costumo dizer a
minha filha.

Saudações,
PJMS


Em 4 de julho de 2017 12:56, Carlos Gomes <cgomes...@gmail.com> escreveu:

> Olá Max...não seu o seu histórico anterior com as Olimpiadas...mas nunca é
> tarde para começar...para o nivel universitário uma ótima referência é o
> livro (PUTNAM BEYOND)  https://libgen.pw/download.php?id=10688 . Uma
> outra boa dica é o proprio site da OBM onde você encontra a revista EUREKA
> e também as provas anteriores.
>
> Um grande abraço, Cgomes.
>
> Em 4 de julho de 2017 10:23, Max Alexandre <maxmalexan...@gmail.com>
> escreveu:
>
>> Olá, pesoal!
>>
>> Sou iniciante na área de olimpíadas. Faço faculdade na área da saúde,
>> então não verei muito calculo ao longo do curso. Mas estou realmente
>> instigado a resolver problemas de matemática. Já até me inscrevi na OBM e
>> na OMERJ. Por isso, peço encarecidamente sugestões de materias voltados a
>> olimpíadas de matemática de nível universitário visando uma preparação mais
>> sólida pras competições.
>>
>> Desde já, agradeço.
>>
>> --
>> Esta mensagem foi verificada pelo sistema de antivírus e
>> acredita-se estar livre de perigo.
>
>
>
> --
> Esta mensagem foi verificada pelo sistema de antivírus e
> acredita-se estar livre de perigo.
>


-- 
Esta mensagem foi verificada pelo sistema de antivírus e
acredita-se estar livre de perigo.

-- 
Esta mensagem foi verificada pelo sistema de antiv�rus e
 acredita-se estar livre de perigo.



[obm-l] Re: [obm-l] Sugestão de material para OBM

2017-07-04 Por tôpico Carlos Gomes
Olá Max...não seu o seu histórico anterior com as Olimpiadas...mas nunca é
tarde para começar...para o nivel universitário uma ótima referência é o
livro (PUTNAM BEYOND)  https://libgen.pw/download.php?id=10688 . Uma outra
boa dica é o proprio site da OBM onde você encontra a revista EUREKA e
também as provas anteriores.

Um grande abraço, Cgomes.

Em 4 de julho de 2017 10:23, Max Alexandre 
escreveu:

> Olá, pesoal!
>
> Sou iniciante na área de olimpíadas. Faço faculdade na área da saúde,
> então não verei muito calculo ao longo do curso. Mas estou realmente
> instigado a resolver problemas de matemática. Já até me inscrevi na OBM e
> na OMERJ. Por isso, peço encarecidamente sugestões de materias voltados a
> olimpíadas de matemática de nível universitário visando uma preparação mais
> sólida pras competições.
>
> Desde já, agradeço.
>
> --
> Esta mensagem foi verificada pelo sistema de antivírus e
> acredita-se estar livre de perigo.

-- 
Esta mensagem foi verificada pelo sistema de antiv�rus e
 acredita-se estar livre de perigo.



Re: [obm-l] boatos sobre elon lages lima

2017-05-23 Por tôpico Carlos Gomes
Infelizmente eh verdade. Foi agora em maio.

Cgomes.

Em 24 de mai de 2017 02:19, "Israel Meireles Chrisostomo" <
israelmchrisost...@gmail.com> escreveu:

> É verdade que o Elon morreu?Fiquei chocado com essa notícia, o pessoal
> aqui poderia confirmar a veracidade dessa notícia?
>
> --
> Esta mensagem foi verificada pelo sistema de antivírus e
> acredita-se estar livre de perigo.

-- 
Esta mensagem foi verificada pelo sistema de antiv�rus e
 acredita-se estar livre de perigo.



Re: [obm-l] Probleminha bacana

2017-03-04 Por tôpico Carlos Gomes
É verdade Pedro...eu também tive exatamente o mesmo sentimento que você. É
tipicamente um daqueles enunciados, mal enunciados. É comum alguém pensar
algo e escrever outra coisa! Nesses caso tento passar para o outro lado e
tentar imaginar o que se passava na cabeça de que criou o problema. Dessa
forma eu supus  que  que quando ele diz "uniforme" ele queira dizer  que
tem intervalos de tempos iguais a probabilidade de se pescar um peixe seja
a mesma. Mas você tem razão, rigorosamente o enunciado precisaria ser
melhor, aliás, ser posto de uma forma correta. Mas acredito fortemente que
era isso que se passava na cabeça de que elaborou.

Em 3 de março de 2017 22:10, Pedro José <petroc...@gmail.com> escreveu:

> Boa noite!
>
> Não compreendi o problema. Para mim há uma curva de distribuição de
> probabilidade.
> Portanto não há como aplicar conceito de modelo discreto. Mas sim integral.
> Também, não entendi o que significa probabilidade uniforme.
>
>
> Saudações,
> PJMS
>
>
>
>
>
> Em 3 de março de 2017 11:45, Carlos Gomes <cgomes...@gmail.com> escreveu:
>
>> Ola Mauricio,
>>
>> Eu pensei assim:
>>
>> seja p a probabilidade de pegar pelo menos um peixe em meia hora (que é o
>> aue você quer  achar!). Assim a probabilidade de nao pegar nenhum peixe em
>> meia hora é 1-p.
>>
>> Como a probabilidade de pegar pelo menos um peixe em uma hora é 0,64,
>> segue que a probabilidade de nao pegar nenhum peixe em uma hora
>> é1-0,64=0,36.
>>
>> Ora, mas se nao pegou um peixe em uma hora, quer dizer que nao pegou
>> nenhum peixe durante a primeira meia hora e tambem nao pegou nehum peixe
>> durante a segunda meia hora, o que ocorre com probabilidade (1-p)(1-p)
>>
>> Assim, (1-p)^2=0,36  ==> 1-p=0,60  ==> p=0,40 (=40%).
>>
>> Cgomes.
>>
>> Em 3 de março de 2017 14:28, Mauricio de Araujo <
>> mauricio.de.ara...@gmail.com> escreveu:
>>
>>>
>>> Em um determinado lago, a probabilidade de se pegar um peixe é uniforme
>>> e independente ao longo do tempo. Se a probabilidade de você pegar pelo
>>> menos um peixe em uma hora é de 64%, qual é a probabilidade de você pegar
>>> pelo menos um peixe em meia hora?
>>>
>>> 60%
>>>
>>> 40%
>>>
>>> 80%
>>>
>>> 32%
>>>
>>>
>>>
>>> --
>>> Abraços,
>>> Mauricio de Araujo
>>> [oɾnɐɹɐ ǝp oıɔıɹnɐɯ]
>>>
>>>
>>> --
>>> Esta mensagem foi verificada pelo sistema de antivírus e
>>> acredita-se estar livre de perigo.
>>
>>
>>
>> --
>> Esta mensagem foi verificada pelo sistema de antivírus e
>> acredita-se estar livre de perigo.
>>
>
>
> --
> Esta mensagem foi verificada pelo sistema de antivírus e
> acredita-se estar livre de perigo.
>

-- 
Esta mensagem foi verificada pelo sistema de antiv�rus e
 acredita-se estar livre de perigo.



Re: [obm-l] Probleminha bacana

2017-03-03 Por tôpico Carlos Gomes
Ola Mauricio,

Eu pensei assim:

seja p a probabilidade de pegar pelo menos um peixe em meia hora (que é o
aue você quer  achar!). Assim a probabilidade de nao pegar nenhum peixe em
meia hora é 1-p.

Como a probabilidade de pegar pelo menos um peixe em uma hora é 0,64, segue
que a probabilidade de nao pegar nenhum peixe em uma hora é1-0,64=0,36.

Ora, mas se nao pegou um peixe em uma hora, quer dizer que nao pegou nenhum
peixe durante a primeira meia hora e tambem nao pegou nehum peixe durante a
segunda meia hora, o que ocorre com probabilidade (1-p)(1-p)

Assim, (1-p)^2=0,36  ==> 1-p=0,60  ==> p=0,40 (=40%).

Cgomes.

Em 3 de março de 2017 14:28, Mauricio de Araujo <
mauricio.de.ara...@gmail.com> escreveu:

>
> Em um determinado lago, a probabilidade de se pegar um peixe é uniforme e
> independente ao longo do tempo. Se a probabilidade de você pegar pelo menos
> um peixe em uma hora é de 64%, qual é a probabilidade de você pegar pelo
> menos um peixe em meia hora?
>
> 60%
>
> 40%
>
> 80%
>
> 32%
>
>
>
> --
> Abraços,
> Mauricio de Araujo
> [oɾnɐɹɐ ǝp oıɔıɹnɐɯ]
>
>
> --
> Esta mensagem foi verificada pelo sistema de antivírus e
> acredita-se estar livre de perigo.

-- 
Esta mensagem foi verificada pelo sistema de antiv�rus e
 acredita-se estar livre de perigo.



Re: [obm-l] Integral e Derivada

2017-02-09 Por tôpico Carlos Gomes
Ola Anselmo. Tenho sugestoes:

1) Na primeira, \sqrt(1-cosx) < ou = \sqrt(2) pois a expressao
\sqrt(1-cosx) assume o seu maior valor quando cosx=-1. Assim,

\int_1^{\infty} \sqrt{\frac{1-\cos(x)}{x^3}} dx < ou =

\int_1^{\infty} \sqrt{\frac{\sqrt(2)}{x^3}} dx =

\sqrt(2)\int_1^{\infty} \sqrt{\frac{1}{x^3}} dx =

\sqrt(2). lim_{a \to \infty} \int_1^a {\frac{1}{x^3}}^{1/2} dx =

\sqrt(2). lim_{a \to \infty} \int_1^a x^{-3/2} dx =

\sqrt(2). lim_{a \to \infty} [x^{-1/2}/(-1/2) ] =

-2\sqrt(2). lim_{a \to \infty} [ 1/\sqrt{x}]_1^a =

-2\sqrt(2). lim_{a \to \infty} [ /\sqrt{a} - /\sqrt{1} ] =

-2\sqrt(2). [ 0-1 ] = 2\sqrt(2)  <  3.


2) Essa basta aplicar diretamente a formula:

se F(x)=\int_a(x)^b(x) g(x,y)dy , entao

F'(x) = g(x,b(x)).b'(x) - g(x,a(x)).a'(x)  +  \int_a(x)^b(x) d/dx
g(x,y)dy

 [essa ultima derivada que aparece nessa ultima integral e a derivada
parcial em relacao a x]

No caso da sua questao,  a(x)=x , b(x)=\sqrt{x} , g(x,y)= exp(xy^2)/y.

Pronto...faz essas continhas que agora sai...se nao consegir me fala aue
termino p vc, eh que estou num pc com um teclado sem conficuracao em
portugues eh horrivel para digitar...por isso nao pus nenhum acento.

Abraco, Cgomes.








Em 9 de fevereiro de 2017 14:47, Anselmo Alves de Sousa <
starterm...@gmail.com> escreveu:

> Muito grato pela disca, mas ainda não consegui. A primeira, segundo a
> resposta f(x) é integrável no intervalo e não supera 3. Na segunda cheguei
> perto, mas ainda não entendi a parte 'derivando dentro da integral'. Se
> puder resolver, agradeço!
>
> sds,
> Sousa
>
> Em 8 de fevereiro de 2017 21:19, Bernardo Freitas Paulo da Costa <
> bernardo...@gmail.com> escreveu:
>
>> 2017-02-08 16:26 GMT-02:00 Anselmo Alves de Sousa > >:
>> > Solicito auxílio pra resolver:
>> >
>> > 1. \int_1^{\infty} \sqrt{\frac{1-\cos(x)}{x^3}} dx
>>
>> Ela é claramente finita.
>> O *quanto* ela vale, acho que só numericamente; acho que nem com
>> resíduos sai.  E como o integrando nem é diferenciável, vai dar
>> trabalho...
>>
>> > 2. obter a derivada de f(x) = \int_{x}^{\sqrt{x}}\frac{\exp(xy^2)}{y}
>> dy
>>
>> Tem que ter cuidado com a derivada, mas não é difícil.  Você vai ficar
>> com três termos: um derivando no limite inferior, outro no limite
>> superior (! cuidado: regra da cadeia para a sqrt) e depois a derivada
>> dentro da integral.  A "parte boa" é que a derivada dentro da integral
>> fará aparecer um y^2 que permite efetuar analiticamente a integral
>> depois da mudança t = y^2, dt = 2ydy. (antes não daria certo)
>>
>> Abraços,
>> --
>> Bernardo Freitas Paulo da Costa
>>
>> --
>> Esta mensagem foi verificada pelo sistema de antivírus e
>>  acredita-se estar livre de perigo.
>>
>>
>> =
>> Instru�ões para entrar na lista, sair da lista e usar a lista em
>> http://www.mat.puc-rio.br/~obmlistas/obm-l.html
>> =
>>
>
>
> --
> Esta mensagem foi verificada pelo sistema de antivírus e
> acredita-se estar livre de perigo.
>

-- 
Esta mensagem foi verificada pelo sistema de antiv�rus e
 acredita-se estar livre de perigo.



[obm-l] Re: [obm-l] Re: [obm-l] Sistema de equações

2017-02-05 Por tôpico Carlos Gomes
Agora o enunciado faz sentido! Esse problema está resolvido nosso livro
Olimpíadas de Matemática do Estado do Rio Grande do Norte - 1985 - 2007.

Abraço, Cgomes,


Em 4 de fevereiro de 2017 14:35, Pacini Bores 
escreveu:

>
>
>
>
>
> Oi Marcone, errei na digitação : digo 1
> Em 04/02/2017 10:34, Pacini Bores escreveu:
>
>
>
>
> Oi Marcone,
>
> Tome x+y=k e faça y = k-x na segunda equação. Observe que 0 0
> No final coloque (k-2) em evidencia e ficará (k-2).p(x)=0; onde p(x) é um
> polinômio do segundo grau em x que não se anulará  nas observações
> colocadas anteriormente.
>
> Logo k=2 , ok ? Confira as contas.
>
> Abraços
>
> Pacini
>
> Em 03/02/2017 17:47, marcone augusto araújo borges escreveu:
>
> Como nada foi afirmado, x e y devem ser números reais
>
>
>
> Se x^3 - 3x^2 + 5x = 1 e y^2 - 3y^2 + 5y = 5, calcule x+y
>
> --
> Esta mensagem foi verificada pelo sistema de antivírus e
> acredita-se estar livre de perigo.
>
>
> --
> Esta mensagem foi verificada pelo sistema de antivírus e
> acredita-se estar livre de perigo.
>

-- 
Esta mensagem foi verificada pelo sistema de antiv�rus e
 acredita-se estar livre de perigo.



[obm-l] Re: [obm-l] Re: Sistema de equações

2017-02-04 Por tôpico Carlos Gomes
Pera, então a segunda equação é  y^2 - 3y^2 + 5y = 5 ==> -2y^2+5y-5=0?
Nesse caso essa equação não possui raízes reais. Tá estranho Marcone.
Suspeito que há algo digitado errado! Confere aí...mesmo que esteja
digitado asim não significa que necessariamente esteja certo!

Cgomes.

Em 4 de fevereiro de 2017 01:32, marcone augusto araújo borges <
marconeborge...@hotmail.com> escreveu:

> não é um sistema, mas como resolver?
>
> --
> *De:* owner-ob...@mat.puc-rio.br  em nome de
> marcone augusto araújo borges 
> *Enviado:* sexta-feira, 3 de fevereiro de 2017 19:47
> *Para:* obm-l@mat.puc-rio.br
> *Assunto:* [obm-l] Sistema de equações
>
>
> Como nada foi afirmado, x e y devem ser números reais
>
>
> Se x^3 - 3x^2 + 5x = 1 e y^2 - 3y^2 + 5y = 5, calcule x+y
>
> --
> Esta mensagem foi verificada pelo sistema de antivírus e
> acredita-se estar livre de perigo.
>
> --
> Esta mensagem foi verificada pelo sistema de antivírus e
> acredita-se estar livre de perigo.
>

-- 
Esta mensagem foi verificada pelo sistema de antiv�rus e
 acredita-se estar livre de perigo.



[obm-l] Re: [obm-l] Re: Sistema de equações

2017-02-04 Por tôpico Carlos Gomes
Olá Marcone,

 A segunda equação está correta ?  Não seria y^3 - 3y^2 + 5y = 5...

Em 4 de fevereiro de 2017 01:32, marcone augusto araújo borges <
marconeborge...@hotmail.com> escreveu:

> não é um sistema, mas como resolver?
>
> --
> *De:* owner-ob...@mat.puc-rio.br  em nome de
> marcone augusto araújo borges 
> *Enviado:* sexta-feira, 3 de fevereiro de 2017 19:47
> *Para:* obm-l@mat.puc-rio.br
> *Assunto:* [obm-l] Sistema de equações
>
>
> Como nada foi afirmado, x e y devem ser números reais
>
>
> Se x^3 - 3x^2 + 5x = 1 e y^2 - 3y^2 + 5y = 5, calcule x+y
>
> --
> Esta mensagem foi verificada pelo sistema de antivírus e
> acredita-se estar livre de perigo.
>
> --
> Esta mensagem foi verificada pelo sistema de antivírus e
> acredita-se estar livre de perigo.
>

-- 
Esta mensagem foi verificada pelo sistema de antiv�rus e
 acredita-se estar livre de perigo.



[obm-l] Re: [obm-l] Enc: Problema (uma função)

2017-01-25 Por tôpico Carlos Gomes
Ola Marcone, (desculpe-me pela falta dos acentos...estou com um teclado
estranho...)

Uma maneira para que ocorra f(f(x))=x eh eh q f seja igual a sua inversa.
Ora, como f(x)=(ax+b)/(cx+d), segue que f^{-1}(x)=(-dx+b)/(cx-a).  para que
f e sua inversa fosse iguais bastaria que d=-a, o que implicaria em
f(x)=(ax+b)/(cx-a). Assim,

f(19)=19  ==> 19=(a.19+b)/(c.19-a) ==> 19^2c-19a=19a+b
f(97)=97  ==> 97=(a.97+b)/(c.97-a) ==> 97^2c-97a=97a+b

subtraindo membro a membro, obtemos: a=58c

substituindo em 19^2c-19a=19a+b, segue que b=-1843c

Finalmente,

 f(x)=(ax+b)/(cx-a)=(58cx-1843c)/(cx-58c)=(58x-1843)/(x-58)


Por fim, o unico numero que nao pertencece a imagem da f eh o mesmo unico
numero que nao esta no dominio da f^{-1} , visto que a imagem de uma funcao
bijetiva eh igual ao dominio da sua inversa. Ora, como

f^{-1}(x)=f(x)=(58x-1843)/(x-58)

segue que o dominio da f^{-1}  e eh R-{58}, que pr sua vez eh igual ao
conjunto imagen da f, relevando, pois que 58 eh o unico numero real que nao
pertence ao conjunto imagem da funcao f.

Cgomes.



Em 25 de janeiro de 2017 17:12, marcone augusto araújo borges <
marconeborge...@hotmail.com> escreveu:

>
>
>
> --
> *De:* marcone augusto araújo borges 
> *Enviado:* quarta-feira, 25 de janeiro de 2017 00:59
> *Para:* obm-l@mat.puc-rio.br
> *Assunto:* Problema (uma função)
>
>
> Seja f : R--> R - {- d/c} uma função definida por f(x) = (ax+b)/(cx+d),
> onde a,b,c,d  E  R - .
>
> Sabendo que f(19) = 19, f(97) = 97 e f(f(x)) = x para todo x, determine o
> único número que
>
> não pertence à imagem de f
>
> --
> Esta mensagem foi verificada pelo sistema de antivírus e
> acredita-se estar livre de perigo.
>

-- 
Esta mensagem foi verificada pelo sistema de antiv�rus e
 acredita-se estar livre de perigo.



[obm-l] Re: [obm-l] Re: [obm-l] Re: [obm-l] Qual a maior potência?

2017-01-16 Por tôpico Carlos Gomes
Bacana Esdras!

A solução do Bruno também é correta,as ele cometeu um errinho na digitação
no começo

5^3=125<128=2^7 ==> (5^3)^14<(2^7)^14 ==> 5^42 <2^98=4^49

mas é bacana tb!

Cgomes.





Em 16 de janeiro de 2017 18:25, Bruno Visnadi 
escreveu:

> 5^3 < 128 = 4^3.5
>
> Então, 4^53/5^44 = 4^53/(5^2)*(5^42) > 4^53/25*4^49 = 4^4/25 > 1
> Portanto, 4^53 > 5^44
>
> Em 16 de janeiro de 2017 15:01, Esdras Muniz 
> escreveu:
>
>> 4^53 = 2^106 > 2^105 = (2^7)^15 = (128)^15 > 125^15 = 5^45 > 5^44.
>>
>> Em 16 de janeiro de 2017 13:14, Douglas Oliveira de Lima <
>> profdouglaso.del...@gmail.com> escreveu:
>>
>>> Olá amigos , gostaria de uma ajuda pra um raciocínio diferente, por log
>>> eu já fiz.
>>>
>>> Qual a maior potência? 4^53 ou 5^44.
>>>
>>> --
>>> Esta mensagem foi verificada pelo sistema de antivírus e
>>> acredita-se estar livre de perigo.
>>
>>
>>
>>
>> --
>> Esdras Muniz Mota
>> Mestrando em Matemática
>> Universidade Federal do Ceará
>>
>>
>>
>> --
>> Esta mensagem foi verificada pelo sistema de antivírus e
>> acredita-se estar livre de perigo.
>>
>
>
> --
> Esta mensagem foi verificada pelo sistema de antivírus e
> acredita-se estar livre de perigo.
>

-- 
Esta mensagem foi verificada pelo sistema de antiv�rus e
 acredita-se estar livre de perigo.



Re: [obm-l] Re: Geometria

2016-09-10 Por tôpico Carlos Gomes
Olá Jeferson,

Como não dá para colocar a figura aqui vou falar...Considere o triângulo
ABC com com o segmento AB na horizontal e A a esquerda de  B. Ponha o
vértice C no topo do triângulo e o ponto D no interior do triângulo ABC
satisfazendo as condições do enunciado. Ao por os ângulos citados no
enunciado podemos perceber que o triângulo ABC é isósceles de base AB.
Assim, CA=CB=a. Agora observe que os triângulos ACD e BCD tem o lado CD em
comum. Denotando a medida do ângulo DCB por x, segue que as medidas dos
ângulos dos triângulos ADC são 40°-x, 120°+x e 20° e as medidas dos ângulos
do triângulo BCD são 40°, 140°-x e x. Aplicando a lei dos senos nesses dois
triângulos segue que

no triângulo BCD: a/sen(140°-x)=CD/sen40°  ==> a/CD=sen(140°-x)/sen40°

no triângulo ACD: a/sen(120°+x)=CD/sen20°  ==> a/CD=sen(120°+x)/sen20°

igualando-se as duas expressões anteriores, segue que
sen(140°-x)sen20°=sen(120°+x)sen40°

sen(140°-x)sen20°=sen(120°+x)2.sen20°.cos20° ==>

sen(140°-x)=2sen(90°+30°+x)cos20° ==>

sen(140°-x)=2sen(90°+30°+x)cos20° ==>

 sen(140°-x)=2cos(30°+x)cos20°.

Nesse ponto pensei num x que pudesse eliminar esse fator 2. Como
cos60°=1/2, um x possível seria 30°. E de fato x=-30° é solução, pois

sen(140°-30°)=2.cos(30°+30°)cos20° ==> sen110°=2.1/2.sen70° ==>
sen110°=sen70°, o que é verdade.

Mas a pergunta é a medida do ângulo BDC. No triângulo BCD, segue que

m(BDC)+x+40°=180°  ==> m(BDC)+30°+40°=180° ==> m(BDC)=110°.

Cgomes.







Em 10 de setembro de 2016 17:34, Jeferson Almir 
escreveu:

> Olá pessoa queria uma ajuda nessa questão
>
> A figura em anexo mostra um triângulo *ABC*. *D* é um ponto interior onde
> a medida dos ângulos *CAD*, *ABD*, *CBD*, e *BAD* são 20º, 30º, 40º e 50º
> , respectivamente. Encontre a medida do ângulo *BDC*.
>
> Em 28 de agosto de 2016 18:31, Jeferson Almir 
> escreveu:
>
>> Olá pessoa queria uma ajuda nessa questão
>>
>> A figura em anexo mostra um triângulo *ABC*. *D* é um ponto interior
>> onde a medida dos ângulos *CAD*, *ABD*, *CBD*, e *BAD* são 20º, 30º, 40º
>> e 50º , respectivamente. Encontre a medida do ângulo *BDC*.
>>
>>
>>
>
> --
> Esta mensagem foi verificada pelo sistema de antivírus e
> acredita-se estar livre de perigo.

-- 
Esta mensagem foi verificada pelo sistema de antiv�rus e
 acredita-se estar livre de perigo.



[obm-l] Re: [obm-l] Equação Trigonométrica

2016-09-10 Por tôpico Carlos Gomes
Olá Ricardo você está certo!

Em 10 de setembro de 2016 14:31, Ricardo Leão 
escreveu:

> Olá amigos,
> Eu tenho uma dúvida em relação ao seguinte enunciado:
>
> Sendo x medida em radianos, com 0 <= x <= 2pi, a soma de todas as raízes
> da equação cos² 2x = sen² x é igual a:
>
> a) 3pi/2   c) 3pi e) 6pi
> b) 2pi  d) 4pi
>
> De acordo com o gabarito oficial a resposta é Item B.
>
> Mas de acordo com meus cálculos x = { pi/6, 5pi/6, 7pi/6, 11pi/6, pi/2,
> 3pi/2} com soma das raízes igual a 6pi.
>
> Por favor, algum colega poderia tirar essa dúvida ?
>
> --
> Esta mensagem foi verificada pelo sistema de antivírus e
> acredita-se estar livre de perigo.

-- 
Esta mensagem foi verificada pelo sistema de antiv�rus e
 acredita-se estar livre de perigo.



[obm-l] Re: [obm-l] Re: [obm-l] Re: [obm-l] Re: Teoria dos números

2016-08-31 Por tôpico Carlos Gomes
Beleza Israel!

Em 29 de agosto de 2016 21:19, Israel Meireles Chrisostomo <
israelmchrisost...@gmail.com> escreveu:

> Me adicione aos seus círculos que vou te mandar um email.
>
> Em 29 de agosto de 2016 21:18, Israel Meireles Chrisostomo <
> israelmchrisost...@gmail.com> escreveu:
>
>> Afinal já tenho vc no facebook ehehehe mas vc quase não está online!
>>
>> Em 29 de agosto de 2016 21:18, Israel Meireles Chrisostomo <
>> israelmchrisost...@gmail.com> escreveu:
>>
>>> Muito obrigado professor Carlos Gomes!Vamos nos falando!Posso te
>>> adicionar no facebook?Lá taçvez nós poderemos nos comunicar melhor!
>>>
>>> Em 29 de agosto de 2016 11:32, Carlos Gomes <cgomes...@gmail.com>
>>> escreveu:
>>>
>>>> Olá Israel, de longe não sou especilista em Teoria dos números, mas sou
>>>> professor da disciplina na graduação e já estudei e apresentei  várias
>>>> vezes para meus alunos aqui na UFRN a demonstração clássica da
>>>> irracionalidade do pi  nos cursos de Teoria dos números. Se vc quiser posso
>>>> tentar ler e quem sabe até ajudar com a sua demonstração da irracionalidade
>>>> do pi. Vou te mandar uns slides que fiz sobre o assunto com a clássica
>>>> demonstração feita pelo prof. Ivan Niven na American Mathematical Monthly
>>>> nos anos 60 se não me engano.
>>>>
>>>> abraço, Cgomes.
>>>>
>>>> Em 28 de agosto de 2016 22:44, Israel Meireles Chrisostomo <
>>>> israelmchrisost...@gmail.com> escreveu:
>>>>
>>>>> Meus professores estão todos ocupados, e não podem me ajudar
>>>>>
>>>>> Em 28 de agosto de 2016 22:43, Israel Meireles Chrisostomo <
>>>>> israelmchrisost...@gmail.com> escreveu:
>>>>>
>>>>>> Tem alguém aqui que é especialista em Teoria dos Números e que esteja
>>>>>> disposto a me ajudar a corrigir uma demonstração que fiz agora pouco, 
>>>>>> cujo
>>>>>> objetivo era provar a irracionalidade de pi?Espero que seja alguém muito
>>>>>> paciente e com muita vontade de ajudar, gostaria de escrever um artigo e
>>>>>> preciso de alguma boa alma para me dar uma força.
>>>>>>
>>>>>
>>>>>
>>>>> --
>>>>> Esta mensagem foi verificada pelo sistema de antivírus e
>>>>> acredita-se estar livre de perigo.
>>>>>
>>>>
>>>>
>>>> --
>>>> Esta mensagem foi verificada pelo sistema de antivírus e
>>>> acredita-se estar livre de perigo.
>>>
>>>
>>>
>>
>
> --
> Esta mensagem foi verificada pelo sistema de antivírus e
> acredita-se estar livre de perigo.
>

-- 
Esta mensagem foi verificada pelo sistema de antiv�rus e
 acredita-se estar livre de perigo.



[obm-l] Re: [obm-l] Generalização de desigualdade

2016-08-29 Por tôpico Carlos Gomes
Olá Israel,

Quem são o x, y e z? São reais positivos? Tem algum significado geométrico
no triângulo?

Em 29 de agosto de 2016 10:51, Israel Meireles Chrisostomo <
israelmchrisost...@gmail.com> escreveu:

> Sejam [ABC] a área de um triângulo agudo e a,b,c seus lados, então vale:
>
> a²x+b²y+c²z>=4[ABC]sqrt{xy+xz+yz}
>
> Como generalizar essa desigualdade para outros tipos de triângulo?
> Eu consigo prová-la para triângulos agudos, usando a desigualdade de
> Jensen e a convexidade da tangente no intervalo (0,pi/2).Alguém pode me
> ajudar?
>
> --
> Esta mensagem foi verificada pelo sistema de antivírus e
> acredita-se estar livre de perigo.

-- 
Esta mensagem foi verificada pelo sistema de antiv�rus e
 acredita-se estar livre de perigo.



[obm-l] Re: [obm-l] Re: Teoria dos números

2016-08-29 Por tôpico Carlos Gomes
Olá Israel, de longe não sou especilista em Teoria dos números, mas sou
professor da disciplina na graduação e já estudei e apresentei  várias
vezes para meus alunos aqui na UFRN a demonstração clássica da
irracionalidade do pi  nos cursos de Teoria dos números. Se vc quiser posso
tentar ler e quem sabe até ajudar com a sua demonstração da irracionalidade
do pi. Vou te mandar uns slides que fiz sobre o assunto com a clássica
demonstração feita pelo prof. Ivan Niven na American Mathematical Monthly
nos anos 60 se não me engano.

abraço, Cgomes.

Em 28 de agosto de 2016 22:44, Israel Meireles Chrisostomo <
israelmchrisost...@gmail.com> escreveu:

> Meus professores estão todos ocupados, e não podem me ajudar
>
> Em 28 de agosto de 2016 22:43, Israel Meireles Chrisostomo <
> israelmchrisost...@gmail.com> escreveu:
>
>> Tem alguém aqui que é especialista em Teoria dos Números e que esteja
>> disposto a me ajudar a corrigir uma demonstração que fiz agora pouco, cujo
>> objetivo era provar a irracionalidade de pi?Espero que seja alguém muito
>> paciente e com muita vontade de ajudar, gostaria de escrever um artigo e
>> preciso de alguma boa alma para me dar uma força.
>>
>
>
> --
> Esta mensagem foi verificada pelo sistema de antivírus e
> acredita-se estar livre de perigo.
>

-- 
Esta mensagem foi verificada pelo sistema de antiv�rus e
 acredita-se estar livre de perigo.



[obm-l] Re: [obm-l] Re: [obm-l] [obm-l] Simulado ITA (Equação Modular)

2016-08-22 Por tôpico Carlos Gomes
Uma boa alternativa é esboçar as representações gráficas das funções
f(x)=||x+1|-2 e g(x)=sqrt{x+4} (que são relativamente simples de esboçar) e
ver que há 4 pontos de interseção; um entre -4 e -3, outros dois entre -2 e
0 e mais um entre 3 e 4.

Abraço, Cgomes.

Em 22 de agosto de 2016 16:58, Ralph Teixeira  escreveu:

> Confira as suas contas -- cada uma daquelas 4 equacoes tem uma raiz real
> valida.
>
> Abraco, Ralph.
>
>
> 2016-08-22 16:38 GMT-03:00 Ricardo Leão :
>
>> Olá amigos,
>>
>> Eu gostaria que algum amigo corrigisse a solução que eu desenvolvi para o
>> seguinte problema envolvendo módulo:
>>
>> (Enunciado) O numero de soluções reais da equação | |x+1| - 2 | =
>> \sqrt{x+4} é:
>>
>> a) 0 b) 1 c) 2  d) 3   e) 4
>>
>> (MINHA SOLUÇÃO):
>>
>> |x+1| = 2 + \sqrt{x+4} ou  |x+1| = 2 - \sqrt{x+4}
>>
>> x + 1 = 2 + \sqrt{x+4}   x + 1 = 2 - \sqrt{x+4}
>> ouou
>> x + 1 = -2 - \sqrt{x+4}  x + 1 = -2 + \sqrt{x+4}
>>
>> Eu chequei e se eu não estiver enganado, o número de soluções é zero.
>>
>> Mas de acordo com o gabarito oficial o resultado é 4(item E).
>>
>> Eu agradeço muito se alguém me ajudar com essa questão.
>>
>> --
>> Esta mensagem foi verificada pelo sistema de antivírus e
>> acredita-se estar livre de perigo.
>
>
>
> --
> Esta mensagem foi verificada pelo sistema de antivírus e
> acredita-se estar livre de perigo.
>

-- 
Esta mensagem foi verificada pelo sistema de antiv�rus e
 acredita-se estar livre de perigo.



[obm-l] Re: [obm-l] Norma e módulo de um vetor

2016-08-10 Por tôpico Carlos Gomes
Olá Pedro,

A noção de norma é a extensão natural da noção de "módulo" definida para os
números reais (ou distância até a origem). Dado um espaço vetorial V sobre
o corpo R (dos números reais) uma norma é uma aplicação || . ||:V --> R que
goza das seguintes propriedades:

N1) ||v|| >0, se v é diferente de zero e ||v||=0 se, e somente se, v=0.
N2) ||a.v||=|a|.||v||, para todo a real e v pertencente a V.
N3) ||u+v|| < ou = ||u||+||v||, para quaisquer u e v em V.

Note que:

em N2) é conveniente usarmos duas barras para a norma de v para não
confundirmos com as barras simples em |a|, que significam apenas o módulo
do número real a. Mas isso é apenas uma opção do autor. Se usássemos apenas
barras simples para representar a norma de um vetor teríamos em  N2)
|a.v|=|a|.|v|, o que não seria conveniente pois estaríamos usando a mesma
notação para duas coisas diferentes, a saber: |a| para o módulo do número
real a e |v| para a norma do vetor v, por isso é conveniente usar duas
barras para a norma de um vetor. No caso específico dos números complexos
geralmente utiliza-se barras simples para representar a sua norma, que é
|z|=(a^2+b^2)^{1/2}, quando z=a+b.i.

Note que se z=a+b.i, então ||z||=a^2+b^2 não é (no sentido da definição
acima) uma norma no R-espaço vetorial dos números complexos pois N3) não
seria satisfeita nesse caso, por exemplo

z=3+4.i  e w=4+3.i teríamos  ||z||=3^2+4^2=25  e ||w||=4^2+3^2=25

z+w=7+7.i  ==> ||z+w||=7^2+7^2=98

ou seja, nesse caso ||z+w||>||z||+||w||.

Assim não é correto dizer que a aplicação || . || : C --> R dada por
||z||=a^2+b+2, quando z=a+b.i não é uma norma no R-espaço vetorial C dos
números complexos.

Resumindo colocamos duas barras para não confundir com o módulo dos números
reais e  ||z||=a^2+b+2 não é uma norma nesse sentido. (Nesse sentido o FME
não está correto em chamar isso de norma)

Obs. Por outro lado num outro contexto (na Teoria algébrica dos números),
define-se a norma de um número complexo a+b.i por N(a+b.i)=a^2+b^2, como
faz o FME, mas nesse novo contexto, apesar do mesmo nome "norma" isso não é
uma norma no sentido que definimos no início. (Nesse sentido o FME está
correto, apesar de que esse não é o contexto a que ele se refere na ocasião
em que ele define o que ele chama de norma.

Espero ter ajudado, Cgomes.



Em 10 de agosto de 2016 20:08, Pedro Henrique 
escreveu:

> Boa noite,
> Estava eu ontem lendo um livro de Álgebra Linear e me deparei com uma
> definição que me causou grandes intrigas, o livro definia norma de um vetor
> bidimensional como sendo ||v|| = sqrt(v1^2 + v2^2). Automaticamente me
> lembrei de minhas aulas de números complexos. Peguei o livro Fundamentos da
> Matemática Elementar e fui em busca da definição de módulo, que por sinal
> era a mesma de norma do livro de Álgebra, |z| = sqrt(a^2 + b^2), e para a
> minha surpresa, o FME definia norma como sendo (a^2 + b^2) - sem a raiz
> quadrada -. Então, qual seria a diferença de módulo e norma de um vetor? Já
> vi em alguns lugares que eles são a mesma coisa, mas se são a mesma coisa,
> pq um é definido com somente um traço na vertical de cada lado |z| enquanto
> o outro possui dois ||v||?
> Desde já agradeço a ajuda.
>
> Obs: O livro de Álgebra é a 3 edição do livro de Howard Anton, traduzido
> da Drexel University.
>
> --
> Esta mensagem foi verificada pelo sistema de antivírus e
> acredita-se estar livre de perigo.

-- 
Esta mensagem foi verificada pelo sistema de antiv�rus e
 acredita-se estar livre de perigo.



[obm-l] Re: [obm-l] Re: [obm-l] Re: [obm-l] Re: [obm-l] Dúvida sobre a Obm U

2016-07-26 Por tôpico Carlos Gomes
Um bom livro é Razvan Gelca, Titu Andreescu-Putnam and Beyond (2007)

Cgomes.

Em 26 de julho de 2016 08:57, Otávio Araújo 
escreveu:

> Não, onde posso conseguir? e do que ela trata?
>
> Em 25 de julho de 2016 11:32, Carlos Victor 
> escreveu:
>
>>
>>
>>
>> Oi Otávio,
>>
>> Você já viu a Revista Matemática Universitária da SBM ?
>>
>> Em 25/07/2016 10:09, Otávio Araújo escreveu:
>>
>>
>>
>> Pois é, se algum professor com experiência em olimpíadas, como o Nicolau
>> por exemplo, respondesse minha pergunta seria de grande ajuda
>>
>> Em 24 de jul de 2016, às 23:25, Israel Meireles Chrisostomo <
>> israelmchrisost...@gmail.com> escreveu:
>>
>> Boa pergunta, eu também tenho interesse em participar da OBM U e
>> gostaria de umas dicas
>>
>> Em 16 de julho de 2016 13:29, Otávio Araújo 
>> escreveu:
>>
>>> Galera, gostaria que vocês me dessem dicas de o que estudar, como
>>> estudar e por quais livros e materiais estudar para a prova da Obm nível
>>> universitário...
>>> Estou muito interessado em participar, mas fico meio confuso por onde
>>> estudar...
>>> Por favor me ajudem
>>> --
>>> Esta mensagem foi verificada pelo sistema de antivírus e
>>> Â acredita-se estar livre de perigo.
>>>
>>>
>>> =
>>> Instruções para entrar na lista, sair da lista e usar a lista em
>>> http://www.mat.puc-rio.br/~obmlistas/obm-l.html
>>> =
>>>
>>
>> --
>> Esta mensagem foi verificada pelo sistema de antivírus e
>> acredita-se estar livre de perigo.
>>
>>
>> --
>> Esta mensagem foi verificada pelo sistema de antivrus e
>> acredita-se estar livre de perigo.
>>
>>
>> --
>> Esta mensagem foi verificada pelo sistema de antivírus e
>> acredita-se estar livre de perigo.
>>
>
>
> --
> Esta mensagem foi verificada pelo sistema de antivírus e
> acredita-se estar livre de perigo.
>

-- 
Esta mensagem foi verificada pelo sistema de antiv�rus e
 acredita-se estar livre de perigo.



[obm-l] Re: [obm-l] [obm-l] Números Complexos

2016-07-10 Por tôpico Carlos Gomes
Olá Daniel,

vc faz assim,

Ora, como w/z=u/w=i, segue que w=i.z e u=i.w. Assim,

u=i.w=i.(i.z)=i^2.z=-1.z=-z ==> z=-u , ou seja, z é o posto de u.
(Alternativa "a")

Abraco, Cgomes.

Em 10 de julho de 2016 13:04, Daniel Rocha 
escreveu:

> Alguém poderia, por favor, solucionar o problema abaixo:
>
> Os números complexos z, w, u são tais que w/z = u/w = i (i é a unidade
> imaginária). É correto afirmar que:
>
> a) z é oposto de u.
> b) z é o conjugado de u.
> c) z é o quadrado de u.
> d) z é igual a u.
> e) z é igual a u + w.
>
>
> --
> Esta mensagem foi verificada pelo sistema de antivírus e
> acredita-se estar livre de perigo.

-- 
Esta mensagem foi verificada pelo sistema de antiv�rus e
 acredita-se estar livre de perigo.



[obm-l] Re: [obm-l] Resto da Divisão por 6

2016-07-07 Por tôpico Carlos Gomes
Olá Marcos...vamos lá...(Vou usar "=" para representar congruente. Como
8=2(mod6) podemos tocar os "8" por 2. Além disso perceba que 2^n=2(mod6) se
n é ímpar e 2^n=4(mod6) se n é par. com (n>0). Assim,

8^1=2(mod6)
8^2=2^2=4(mod6)
8^3=2^3=2(mod6)
.
.
.
8^15=2^15=2(mod6)

adicionando membro a membro, segue que

 8^1 + 8^2 + 8^3 + ... + 8^15=2+4+2+...+2=8x2+7x4=44=2(mod6)

portanto o resto da divisão do número  8^1 + 8^2 + 8^3 + ... + 8^15 por 6 é
igual a 2.

Cgomes.

Em 7 de julho de 2016 11:59, Marcos Xavier  escreveu:

> Prezados amigos,
>
> como resolver o seguinte problema:
>
> Qual o resto obtido ao dividirmos 8^1 + 8^2 + 8^3 + ... + 8^15 por 6?
>
> Grato pela ajuda.
>
> Marcos Xavier
>
> --
> Esta mensagem foi verificada pelo sistema de antivírus e
> acredita-se estar livre de perigo.
>

-- 
Esta mensagem foi verificada pelo sistema de antiv�rus e
 acredita-se estar livre de perigo.



Re: [obm-l] Desigualdade.

2016-06-25 Por tôpico Carlos Gomes
Olá Douglas,

Na maioria dos livros essa desigualdade é usada para definir uma função
convexa, ou seja, uma função f:[x,y] -->R que satisfaz a condição
f(ty+(1-t)x)<=tf(y)+(1-t)f(x) com t E [0,1] é, por definição convexa.

No entanto se a função f:[x,y] --> possuir derivada segunda no intervalo
(x,y), então pode-se mostrar que f(ty+(1-t)x)<=tf(y)+(1-t)f(x) com t E [0,1]
se, e somente se f ''(a)>=0, para todo a E [x,y].

Talvez seja isso que vc quer: supondo que f ''(a)>=0, para todo a E [x,y],
mostrar que f(ty+(1-t)x)<=tf(y)+(1-t)f(x) com t E [0,1]. SE for vc faz
assim:

Determina a fórmula de Taylor de f em torno de x e de y: sendo z=ty+(1-t)x,
com t E [0,1] um ponto genérico do intervalo [x,y], segue que

f(x)=f(z)+f'(z)(x-z)+1/2.f ''(c_1)(x-z)^2, com c_1 E [x,z]
f(y)=f(z)+f'(z)(y-z)+1/2.f ''(c_2)(y-z)^2, com c_2 E [x,z]

multiplicando a primeira por (1-t), a segunda por t a adicionando membro a
membro, segue que

tf(y)+(1-t)f(x)=f(z)+R  (Faça as contas para conferir!)

onde R:=1/2(1-t).f ''(c_1)(x-z)^2+1/2.t.f ''(c_2)(y-z)^2>=0, pois (1-t), t
e f '' são >=0.

Assim,

tf(y)+(1-t)f(x)=f(z)+R>=f(z)=f(ty+(1-t)x)

ou seja, f(ty+(1-t)x)<=tf(y)+(1-t)f(x) com t E [0,1].

Abraço, Cgomes.




Em 25 de junho de 2016 20:55, Douglas Oliveira de Lima <
profdouglaso.del...@gmail.com> escreveu:

> Olá amigos preciso de ajuda na seguinte questão:
>
> Mostrar que f(ty+(1-t)x)<=tf(y)+(1-t)f(x) com t E [0,1] e f sendo convexa.
>
> Obs: Não usar geometria.
>
> Agradeço a ajuda.
>
> Douglas Oliveira
>
> --
> Esta mensagem foi verificada pelo sistema de antivírus e
> acredita-se estar livre de perigo.

-- 
Esta mensagem foi verificada pelo sistema de antiv�rus e
 acredita-se estar livre de perigo.



[obm-l] Re: [obm-l] Re: [obm-l] PDF sobre OLimpíadas

2016-06-24 Por tôpico Carlos Gomes
Acabei de folhear rapidamente, mas mesmo rápido já dá para perceber a
qualidade do seu trabalho Israel. Parabéns, a comunidade olímpica agradece!

Abraço, Cgomes.

Em 24 de junho de 2016 20:13, Mauricio de Araujo <
mauricio.de.ara...@gmail.com> escreveu:

> Israel, muito bom este trabalho!! vou dar uma olhada e, se for o caso,
> sugerirei alguma adequação... parabéns!!
>
> Em 24 de junho de 2016 19:25, Israel Meireles Chrisostomo <
> israelmchrisost...@gmail.com> escreveu:
>
>> Olá pessoal estou compartilhado um PDF que escrevi, acrescentei vários
>> problemas:
>>
>> http://media.wix.com/ugd/3eea37_3049c428c55948f2b8bb069834275f50.pdf
>>
>> Quem tiver alguma sugestão ou correção, por favor envie para o meu email,
>> pois muitas pessoas podem se beneficiar com o acerto ou mesmo se prejudicar
>> com o erro.
>>
>> Obrigado.
>>
>> israelmchrisost...@gmail.com
>>
>> --
>> Esta mensagem foi verificada pelo sistema de antivírus e
>> acredita-se estar livre de perigo.
>
>
>
>
> --
>
> Abraços,
> oɾnɐɹɐ ǝp oıɔıɹnɐɯ
>
>
> --
> Esta mensagem foi verificada pelo sistema de antivírus e
> acredita-se estar livre de perigo.
>

-- 
Esta mensagem foi verificada pelo sistema de antiv�rus e
 acredita-se estar livre de perigo.



[obm-l] Re: [obm-l] Re: [obm-l] Re: [obm-l] Sólido de Revolução

2016-06-18 Por tôpico Carlos Gomes
Você está certo, mas o enunciado precisaria dizer...o triângulo cujos
vértices são esses pontos...isso não está claro no enunciado...um enunciado
precisa ser claro!

Cgmes
Em 18 de jun de 2016 20:38, "Alexandre Antunes" <
prof.alexandreantu...@gmail.com> escreveu:

>
> Boa noite,
>
> Apesar do enunciado estranho, parece que ele "gera" um triângulo sim!
> Tentem fazer o esboço do gráfico e vejam se eu errei algo!
> Além disso, a resposta desse volume é 4.Pi ... Vejam o meu raciocínio:
>
> 1) de f(x) = sqrt(4 - x^2) de [-2,2] em [0,2], temos:
> y = sqrt(4-x^2)  ==>  y^2 = 4 - x^2  ==> x^2 +y^2 = 4 (circunferência
> de raio igual a 2)
> no Domínio: [-2,2] e Imagem: [0,2], que nos dá a parte da
> circunferência acima do eixo x;
>
> 2) da informação "|x| = 1", temos as retas x = 1 e x = -1
> A interseção dessas retas com o gráfico definido em (1), nos dá os
> pontos: (-1, sqrt(3)) e (1, sqrt(3));
>
> 3) da informação "Considere a origem e os pontos (x,y) do gráfico da
> função tais que |x| = 1", temos os três pontos que "geram" o triângulo: (0,
> 0), (-1, sqrt(3)) e (1, sqrt(3)).
>
> 4) Agora, usando a nossa "imaginação", ao rotacionar esse triângulo em
> torno "dos eixo das abscissas", temos um sólido de revolução!!!
>
> 5) Para calcular esse volume, podemos pensar esse sólido como um cilindro
> "menos" dois cones (um de cada lado), dessa forma
>
> Vsol_rev = Vcil - 2.Vcone = Pi.[sqrt(3)]^2.(1) - 2. {Pi.[sqrt(3)]^2 .
> (1)}/3 = 6.Pi - 2.Pi = 4.Pi
>
> Obs: Peço desculpas em eventuais erros na digitação dos cálculos, mas os
> colegas entendem como é difícil fazer isso por aqui!!!
>
> Fico no aguardo dos comentários.
>
>
>
>
>
> Atenciosamente,
>
> Prof. Msc. Alexandre Antunes
> www alexandre antunes com br
>
> Em 17 de junho de 2016 23:39, Carlos Gomes <cgomes...@gmail.com> escreveu:
>
>> Daniel primeiro não há triângulo algum e se for o sólido natural q tem q
>> ser o volume seria 4pi/3.
>> Em 17 de jun de 2016 23:21, "Daniel Rocha" <daniel.rocha@gmail.com>
>> escreveu:
>>
>>> Alguém, por favor, poderia solucionar a questão abaixo:
>>>
>>> Dada a função real definida por f(x) = sqrt(4 - x^2) de [-2,2] em [0,2].
>>> Considere a origem e os pontos (x,y) do gráfico da função tais que |x| = 1.
>>> A rotação do triângulo assim obtido, em torno dos eixo das abscissas, gera
>>> um sólido de volume:
>>>
>>> Gabarito: 4Pi
>>>
>>> --
>>> Esta mensagem foi verificada pelo sistema de antivírus e
>>> acredita-se estar livre de perigo.
>>
>>
>> --
>> Esta mensagem foi verificada pelo sistema de antivírus e
>> acredita-se estar livre de perigo.
>>
>
>
> --
> Esta mensagem foi verificada pelo sistema de antivírus e
> acredita-se estar livre de perigo.

-- 
Esta mensagem foi verificada pelo sistema de antiv�rus e
 acredita-se estar livre de perigo.



[obm-l] Re: [obm-l] Sólido de Revolução

2016-06-17 Por tôpico Carlos Gomes
Daniel primeiro não há triângulo algum e se for o sólido natural q tem q
ser o volume seria 4pi/3.
Em 17 de jun de 2016 23:21, "Daniel Rocha" 
escreveu:

> Alguém, por favor, poderia solucionar a questão abaixo:
>
> Dada a função real definida por f(x) = sqrt(4 - x^2) de [-2,2] em [0,2].
> Considere a origem e os pontos (x,y) do gráfico da função tais que |x| = 1.
> A rotação do triângulo assim obtido, em torno dos eixo das abscissas, gera
> um sólido de volume:
>
> Gabarito: 4Pi
>
> --
> Esta mensagem foi verificada pelo sistema de antivírus e
> acredita-se estar livre de perigo.

-- 
Esta mensagem foi verificada pelo sistema de antiv�rus e
 acredita-se estar livre de perigo.



Re: [obm-l] Volume do Comprimido

2016-06-10 Por tôpico Carlos Gomes
Olá Daniel...a resposta é outra!

O volume nesse caso é a soma do volume de um cilíndo mais duas semiesferas,
assim,

V=V(cilindro)+2V(semiesfera)=pi.R^2.h+4/3.pi.R^3=pi.(1/4)^2.2+4.3.pi.(1/4)^3=7.pi/48
cm^3.



Em 10 de junho de 2016 13:50, Daniel Rocha 
escreveu:

> Alguém, por favor, poderia solucionar a questão abaixo:
>
> Considere um comprimido que tem forma cilíndrica, comprimento 2
> centímetros, com hemisférios de diâmetro 0,5 centímetro cada extremidade.
> Qual é o volume desse comprimido (em cm^3) ?
>
> Gabarito: 11Pi/96
>
> --
> Esta mensagem foi verificada pelo sistema de antivírus e
> acredita-se estar livre de perigo.

-- 
Esta mensagem foi verificada pelo sistema de antiv�rus e
 acredita-se estar livre de perigo.



[obm-l] Re: [obm-l] Outra dúvida

2016-06-09 Por tôpico Carlos Gomes
Vamos lá...Daniel o seu enunciado tá estranho não quer conferi-lo para nós?



Em 9 de junho de 2016 19:26, Daniel Rocha 
escreveu:

> Alguém poderia, por favor, solucionar o problema abaixo:
>
> Calculou-se o volume de um cone reto de geratriz 1 e área lateral k. O
> maior valor inteiro que k pode assumir é:
>
> GABARITO: 3
>
>
>
> --
> Esta mensagem foi verificada pelo sistema de antivírus e
> acredita-se estar livre de perigo.

-- 
Esta mensagem foi verificada pelo sistema de antiv�rus e
 acredita-se estar livre de perigo.



[obm-l] Re: [obm-l] Dúvida em Geometria Espacial

2016-06-08 Por tôpico Carlos Gomes
Vc pode fazer assim:

área total = 60 ==> 2.pi.r^2+2.pi.r.h=60  ==>h=(60-2.pi.r^2)/(2.pi.r)   (*)
Por outro lado o volume é

V=pi.r^2.h

substituindo a expressão (*) do h , segue que

V=60r-2.pi.r^3

Fazendo dV/dr=0 (derivada igual a zero para achar os pontos críticos),
segue que

0=60-6.pir^2  ==> r=sqtr(10/pi) (é ponto de máximo, pois
d^2V/dr^2(sqtr(10/pi))<0

substituindo com esse valor de r na expressão (*), segue que
h=2.sqrt(10/pi).

O que revela que r/h=sqtr(10/pi) / 2.sqtr(10/pi)=1/2.

Cgomes.

Em 8 de junho de 2016 19:17, Daniel Rocha 
escreveu:

> Alguém poderia, por favor, solucionar a questão abaixo:
>
> Uma lata de forma cilíndrica, com tampa, deve ser construída com 60 cm^2
> de folha de alumínio. Se r é o raio da base e h é a altura da lata que
> proporcionam o volume máximo, então o valor de r/h é:
>
> GABARITO: 1/2
>
> --
> Esta mensagem foi verificada pelo sistema de antivírus e
> acredita-se estar livre de perigo.

-- 
Esta mensagem foi verificada pelo sistema de antiv�rus e
 acredita-se estar livre de perigo.



[obm-l] Re: [obm-l] Dúvida em Logaritmos

2016-06-06 Por tôpico Carlos Gomes
log[(sqrt 2)^(x-2)] = x ==>
(x-2)log(sqrt 2) = x ==>
x=2log(sqrt2)/(log(sqrt2)-1).

Cgomes.

Em 6 de junho de 2016 19:23, Daniel Rocha 
escreveu:

> Alguém poderia, por favor, solucionar o problema abaixo:
>
> Ache a solução real da equação:
> log[(sqrt 2)^(x-2)] = x
>
> --
> Esta mensagem foi verificada pelo sistema de antivírus e
> acredita-se estar livre de perigo.

-- 
Esta mensagem foi verificada pelo sistema de antiv�rus e
 acredita-se estar livre de perigo.



[obm-l] Re: [obm-l] Re: [obm-l] Re: [obm-l] Dúvida em Geometria Plana

2016-06-05 Por tôpico Carlos Gomes
De nada amigo! Sempre um prazer qdo posso ajudar!

Abraço, Cgomes.

Em 2 de junho de 2016 19:03, Daniel Rocha <daniel.rocha@gmail.com>
escreveu:

> Muito Obrigado, Carlos !!!
>
> Em 2 de junho de 2016 18:54, Carlos Gomes <cgomes...@gmail.com> escreveu:
>
>> Seja x a medida do ângulo BAC. Como o triângulo APQ é isosceles de base
>> AP, segue q a medida do ângulo APQ também é x. Note que o ângulo BQP é
>> externo ao triângulo APQ, portanto, mede x+x=2x. Agora como o triângulo BQP
>> é isosceles de base BQ, segue que o ângulo PBQ também mede 2x. Por fim note
>> que o ângulo BPC é externo ao triângulo  ABP, portanto mede x+2x=3x...como
>> o triângulo BCP também é isosceles de base PC, segue que o ângulo PCB
>> também mede 3x...como o triângulo ABC é isosceles, segue que o ângulo ABC
>> também mede 3x, o que revela q o ângulo PCB mede x. Assim, no triângulo BCP
>> temos que
>> x+3x+3x=π   ==>x=π/7.
>> Em 2 de jun de 2016 18:32, "Daniel Rocha" <daniel.rocha@gmail.com>
>> escreveu:
>>
>>> Olá a todos,
>>>
>>> Alguém poderia, por favor, apresentar os cálculos corretos da seguinte
>>> questão:
>>>
>>> Considere um triângulo ABC isósceles de base BC, e os pontos P e Q tais
>>> que P pertence a AC e Q pertence a AB. Se BC=BP=PQ=QA, a medida do ângulo
>>> do vértice A, em radianos, é:
>>>
>>> GABARITO: Pi/7.
>>>
>>> Eu agradeço a quem apresentar os cálculos corretos.
>>>
>>> --
>>> Esta mensagem foi verificada pelo sistema de antivírus e
>>> acredita-se estar livre de perigo.
>>
>>
>> --
>> Esta mensagem foi verificada pelo sistema de antivírus e
>> acredita-se estar livre de perigo.
>
>
>
> --
> Esta mensagem foi verificada pelo sistema de antivírus e
> acredita-se estar livre de perigo.
>

-- 
Esta mensagem foi verificada pelo sistema de antiv�rus e
 acredita-se estar livre de perigo.



[obm-l] Re: [obm-l] Dúvida em Geometria Plana

2016-06-02 Por tôpico Carlos Gomes
Seja x a medida do ângulo BAC. Como o triângulo APQ é isosceles de base AP,
segue q a medida do ângulo APQ também é x. Note que o ângulo BQP é externo
ao triângulo APQ, portanto, mede x+x=2x. Agora como o triângulo BQP é
isosceles de base BQ, segue que o ângulo PBQ também mede 2x. Por fim note
que o ângulo BPC é externo ao triângulo  ABP, portanto mede x+2x=3x...como
o triângulo BCP também é isosceles de base PC, segue que o ângulo PCB
também mede 3x...como o triângulo ABC é isosceles, segue que o ângulo ABC
também mede 3x, o que revela q o ângulo PCB mede x. Assim, no triângulo BCP
temos que
x+3x+3x=π   ==>x=π/7.
Em 2 de jun de 2016 18:32, "Daniel Rocha" 
escreveu:

> Olá a todos,
>
> Alguém poderia, por favor, apresentar os cálculos corretos da seguinte
> questão:
>
> Considere um triângulo ABC isósceles de base BC, e os pontos P e Q tais
> que P pertence a AC e Q pertence a AB. Se BC=BP=PQ=QA, a medida do ângulo
> do vértice A, em radianos, é:
>
> GABARITO: Pi/7.
>
> Eu agradeço a quem apresentar os cálculos corretos.
>
> --
> Esta mensagem foi verificada pelo sistema de antivírus e
> acredita-se estar livre de perigo.

-- 
Esta mensagem foi verificada pelo sistema de antiv�rus e
 acredita-se estar livre de perigo.



Re: [obm-l] coordenadas do ortocentro

2016-02-21 Por tôpico Carlos Gomes
É na RPM 43 no artigo "Coordenadas para para os Centro do Triângulo"

Se A,B e C são os vértices, H o ortogentro, e a,b e c são as medidas dos
ângulos internos do triângulo, então

H=(A.tga+B.tgb+C.tgc)/(tga+tg+b+tgc)

Cgomes.



Em 21 de fevereiro de 2016 14:04, Luís  escreveu:

> Sauda,c~oes,
>
>
> Sejam os pontos A=(x_A, 1/x_A), B=(x_B, 1/x_B) e C=(x_C, 1/x_C)
>
> vértices do triângulo ABC. Então H=(x_H, 1/x_H).
>
>
> Encontrei a prova deste resultado em diversos sites.
>
>
> Há algum (muito) tempo o Morgado publicou um artigo numa RPM
>
> dando as coordenadas dos centros notáveis do triângulo.
>
>
> Alguém sabe qual o volume ? E quais são as coordenadas do ortocentro ?
>
>
> Obrigado.
>
>
> Luís
>
>
>
> --
> Esta mensagem foi verificada pelo sistema de antivírus e
> acredita-se estar livre de perigo.
>

-- 
Esta mensagem foi verificada pelo sistema de antiv�rus e
 acredita-se estar livre de perigo.



Re: [obm-l] Trigonometria

2015-11-20 Por tôpico Carlos Gomes
Olá Israel, muito bom o seu material. A comunidade olímpica agradece!

Abraço, Cgomes.

Em 20 de novembro de 2015 14:50, Israel Meireles Chrisostomo <
israelmchrisost...@gmail.com> escreveu:

> Olá pessoal, vou deixar um arquivo pdf que escrevi resolvendo problemas em
> trigonometria:
> http://media.wix.com/ugd/3eea37_c5b19270d73a42a9b0068a02d079846c.pdf
> Aqui um vídeo legal em que demonstro a desigualdade
> senA+senB+senC<=3sqrt(3)/2, se A,B,C são ângulos de um triângulo, a
> demonstração é geométrica e interessante, eis aí o vídeo(se puderem deixar
> um comentário lá no vídeo, eu agradeço):
> https://www.youtube.com/watch?edit=vd=ruCqqrMOzCQ
>
> --
> Esta mensagem foi verificada pelo sistema de antivírus e
> acredita-se estar livre de perigo.

-- 
Esta mensagem foi verificada pelo sistema de antiv�rus e
 acredita-se estar livre de perigo.



Re: [obm-l] site com problema bacanas para quem quiser.

2015-11-07 Por tôpico Carlos Gomes
Excelente dica Maurício!  Baytic Way sempre traz problemas muito criativos.
Ótima fonte para a nossa comunidade olímpica!

Abraço, Cgomes.

2015-11-06 14:21 GMT-03:00 Mauricio de Araujo 
:

> http://www.math.olympiaadid.ut.ee/eng/html/index.php?id=bw
>
>
> --
> Abraços
>
> oɾnɐɹɐ ǝp oıɔıɹnɐɯ
>
>
> --
> Esta mensagem foi verificada pelo sistema de antivírus e
> acredita-se estar livre de perigo.

-- 
Esta mensagem foi verificada pelo sistema de antiv�rus e
 acredita-se estar livre de perigo.



Re: [obm-l] Exponencial e polinômios

2015-05-03 Por tôpico Carlos Gomes

Obrigado Artur!

Abraço, Cgomes.

On 03/05/2015 07:24, Artur Costa Steiner wrote:

Uma outra forma de mostrar isto é a seguinte:

Se o polinômio P for constante, só pode haver uma raiz porque a exponencial é 
bijetora.

Se P tiver grau = 1, quando x tende a oo, a exponencial se descola de P, 
mesmo que este também vá para oo. Logo, o conjunto A das raízes de sua equação é limitado 
superiormente. Quando x vai para -oo, a exponencial vai para 0 e P para + ou - oo, de modo 
que as duas curvas se descolam. Assim, A também é limitado inferiormente, logo limitado.

Se A for infinito, então, pelo Teorema de Bolzano Weierstrass, terá um ponto de 
acumulação em R, logo no domínio de ambas as funções. Como ambas são analíticas 
(dadas em todo o R por séries de potências), então coincidem em toda a reta 
real, sendo portanto a mesma função. Mas, pelo que vimos, isto é impossível. 
Logo, só pode haver um número finito de raízes, A é finito.

Se duas funções analíticas em R coincidirem em um conjunto limitado, então são 
a mesma função. Assim, um argumento similar ao anterior mostra, por exemplo, 
que, se P não for constante, a equação P(x) = sin(x) têm um número finito de 
raízes em R.

Isto vale também nos complexos. Assim, se P for um polinômio de coeficientes 
complexos e f(z) = e^(kz), k uma constante complexa não nula, então, em 
subconjuntos limitados de C, f e P igualam-se em um número finito de pontos.

Em subconjuntos infinitos de C, duas funções analíticas pode concordar em um 
uma infinidade de pontos sem serem a mesma função. O conjunto dos pontos em que 
se igualam é sempre enumerável (considerando-se conjuntos finitos como 
enumeráveis).

É interessante observar que, se P não for identicamente nulo, então, em todo o 
C, P e f(z) = e^kz (k não nulo) igualam-se em um conjunto infinito enumerável. 
E em toda reta de C, igualam-se em um número finito de pontos.

Abraços

Artur Costa Steiner


Em 03/05/2015, às 00:50, Carlos Gomes cgomes...@gmail.com escreveu:

Olá amigos,

Será que alguém pode me ajudar com essa?

Mostre que para qualquer polinômio com coeficientes reais p(x)  a equação 
e^x=p(x) tem sempre uma quantidade finita de raízes (evidentemente pode não ter 
raízes, caso em que a quantidade de raízes é 0 e portanto essa quantidade é 
finita inclusive nesse caso).

Abraço, Cgomes.

--
Esta mensagem foi verificada pelo sistema de antivírus e
acredita-se estar livre de perigo.

=
Instruções para entrar na lista, sair da lista e usar a lista em
http://www.mat.puc-rio.br/~obmlistas/obm-l.html
=



--
Esta mensagem foi verificada pelo sistema de antiv�rus e
acredita-se estar livre de perigo.

=
Instru��es para entrar na lista, sair da lista e usar a lista em
http://www.mat.puc-rio.br/~obmlistas/obm-l.html
=


Re: [obm-l] Exponencial e polinômios

2015-05-03 Por tôpico Carlos Gomes

Olá Gugu,

Obrigado pela bela solução!

Abraço, Cgomes.

On 03/05/2015 03:02, g...@impa.br wrote:

   Caro Carlos Gomes,
   Se f(x) é derivável então, pelo teorema do valor médio, entre duas 
raízes de f(x) sempre há uma raiz de f'(x). Assim, se f(x) tem pelo 
menos k raízes então f'(x) tem pelo menos k-1 raízes. Temos que 
f(x)=e^x-p(x) é infinitas vezes derivável. Se p(x) for um polinômio de 
grau n, se derivarmos p(x) n+1 vezes dá 0, e logo se derivarmos f(x) 
n+1 vezes dá e^x, que não tem raiz nenhuma. Portanto, f(x) tem no 
máximo n+1 raízes.

   Abraços,
 Gugu

Quoting Carlos Gomes cgomes...@gmail.com:


Olá amigos,

Será que alguém pode me ajudar com essa?

Mostre que para qualquer polinômio com coeficientes reais p(x) a  
equação e^x=p(x) tem sempre uma quantidade finita de raízes 
(evidentemente pode não ter raízes, caso em que a quantidade de 
raízes é 0 e portanto essa quantidade é finita inclusive nesse caso).


Abraço, Cgomes.

--
Esta mensagem foi verificada pelo sistema de antivírus e
acredita-se estar livre de perigo.

= 


Instruções para entrar na lista, sair da lista e usar a lista em
http://www.mat.puc-rio.br/~obmlistas/obm-l.html
= 






This message was sent using IMP, the Internet Messaging Program.






--
Esta mensagem foi verificada pelo sistema de antiv�rus e
acredita-se estar livre de perigo.

=
Instru��es para entrar na lista, sair da lista e usar a lista em
http://www.mat.puc-rio.br/~obmlistas/obm-l.html
=


[obm-l] Exponencial e polinômios

2015-05-02 Por tôpico Carlos Gomes

Olá amigos,

Será que alguém pode me ajudar com essa?

Mostre que para qualquer polinômio com coeficientes reais p(x)  a 
equação e^x=p(x) tem sempre uma quantidade finita de raízes 
(evidentemente pode não ter raízes, caso em que a quantidade de raízes é 
0 e portanto essa quantidade é finita inclusive nesse caso).


Abraço, Cgomes.

--
Esta mensagem foi verificada pelo sistema de antivírus e
acredita-se estar livre de perigo.

=
Instruções para entrar na lista, sair da lista e usar a lista em
http://www.mat.puc-rio.br/~obmlistas/obm-l.html
=


[obm-l] Exponencial e polinômios

2015-05-02 Por tôpico Carlos Gomes

Olá amigos,

Será que alguém pode me ajudar com essa?

Mostre que para qualquer polinômio com coeficientes reais p(x)  a 
equação e^x=p(x) tem sempre uma quantidade finita de raízes 
(evidentemente pode não ter raízes, caso em que a quantidade de raízes é 
0 e portanto essa quantidade é finita inclusive nesse caso).


Abraço, Cgomes.

--
Esta mensagem foi verificada pelo sistema de antivírus e
acredita-se estar livre de perigo.

=
Instruções para entrar na lista, sair da lista e usar a lista em
http://www.mat.puc-rio.br/~obmlistas/obm-l.html
=


[obm-l] Centro da circunferência

2015-01-06 Por tôpico Carlos Gomes
Olá amigos,

Algum de você pode me ajudar com essa questão:

Seja P um ponto no interior de um círculo tal que existem três cordas que
passam
por P e tem o mesmo comprimento. Prove que P é o centro do círculo.

Grato, Cgomes.

-- 
Esta mensagem foi verificada pelo sistema de antiv�rus e
 acredita-se estar livre de perigo.



[obm-l] Re: [obm-l] Re: [obm-l] Centro da circunferência

2015-01-06 Por tôpico Carlos Gomes
Obrigado pela resposta Esdras, mas ainda não entendi como você garante que
existem pontos A, B e C que distam x de P?

Cgomes.

Em 6 de janeiro de 2015 13:31, Esdras Muniz esdrasmunizm...@gmail.com
escreveu:

 Diga!os que o comprimento das cordas seja l, então P divide l em duas
 partes de comprimentos x e y, assim, x.y seria a potencia de P e  a some de
 x e y seria l. Dai vc tira que existem dois nu!eros x e y que dependem de l
 e P, que são invariantes. Agora sejam A, B e C os pontos da circunferência
 distando x de p, então P seria o circuncentro do triangulo ABC.
 Em 06/01/2015 12:55, Carlos Gomes cgomes...@gmail.com escreveu:

 Olá amigos,

 Algum de você pode me ajudar com essa questão:

 Seja P um ponto no interior de um círculo tal que existem três cordas que
 passam
 por P e tem o mesmo comprimento. Prove que P é o centro do círculo.

 Grato, Cgomes.

 --
 Esta mensagem foi verificada pelo sistema de antivírus e
 acredita-se estar livre de perigo.


 --
 Esta mensagem foi verificada pelo sistema de antivírus e
 acredita-se estar livre de perigo.

-- 
Esta mensagem foi verificada pelo sistema de antiv�rus e
 acredita-se estar livre de perigo.



[obm-l] Triângulo e circunferências

2014-10-09 Por tôpico Carlos Gomes
Olá meus caros...depois de uma longa temporada em off na lista vou postar
uma perguntinha...

Traçamos o círculo T de centro O circunscrito a um triângulo ABC, retângulo
em A de catetos 3 cm e 4 cm. Encontre o raio do círculo W de centro O´,
tangente aos catetos de ABC e interiormente a T

Alguém tem um boa solução?

Abraços, Carlos Gomes.

-- 
Esta mensagem foi verificada pelo sistema de antiv�rus e
 acredita-se estar livre de perigo.



[obm-l] Re: [obm-l] Re: [obm-l] Triângulo e circunferências

2014-10-09 Por tôpico Carlos Gomes
Obrigado Douglas...achei uma outra solução quase agora sem usar inversão...
Mesmo assim muito obrigado pela sua bela solução!

Abraço, Cgomes.

Em 10 de outubro de 2014 00:05, Douglas Oliveira de Lima 
profdouglaso.del...@gmail.com escreveu:

 Então faça uma inversão de polo em A e raio AI sendo I o incentro de ABC,
 vai perceber que o incírculo do ABC é o inverso do círculo cujo o raio
 queremos determinar, assim a resposta será 2.

 Abraços do
 Douglas Oliveira.

 Em 9 de outubro de 2014 21:51, Carlos Gomes cgomes...@gmail.com
 escreveu:

 Olá meus caros...depois de uma longa temporada em off na lista vou postar
 uma perguntinha...

 Traçamos o círculo T de centro O circunscrito a um triângulo ABC,
 retângulo em A de catetos 3 cm e 4 cm. Encontre o raio do círculo W de
 centro O´, tangente aos catetos de ABC e interiormente a T

 Alguém tem um boa solução?

 Abraços, Carlos Gomes.


 --
 Esta mensagem foi verificada pelo sistema de antivírus e
 acredita-se estar livre de perigo.



 --
 Esta mensagem foi verificada pelo sistema de antivírus e
 acredita-se estar livre de perigo.

-- 
Esta mensagem foi verificada pelo sistema de antiv�rus e
 acredita-se estar livre de perigo.



[obm-l] Permutação circular

2012-01-26 Por tôpico Carlos Gomes
Olá amigos...alguém poderia me ajudr com a questão:

De quantas formas distintas 5 casais podem ser dispostos em torno de uma mesa 
circular, supondo que cada marido não fique ao lado da sua respectiva esposa?

(Duas conficurações são consideradas iguais se uma puder ser obtida da outroa 
por um movimento de rotação!)


Obrigado,  Cgomes.

[obm-l] 2^70 + 3^70 eh divisivel por 13

2011-08-31 Por tôpico Carlos Gomes

- Original Message - 
From: Carlos Gomes 
To: obm-l@mat.puc-rio.br 
Sent: Tuesday, August 30, 2011 11:58 PM
Subject: Re: [obm-l] 2^70 + 3^70 eh divisivel por 13


Eu fiz assim, se a e b são inteiros e n é impar 

a+b | a^n+b^n

Assim,

2^70+3^70 = (2^2)^35+(3^2)^35. Ora como 35 é ímpar segue que 2^2+3^2 |  
(2^2)^35+(3^2)^35  ==  13|2^70+3^70 

Cgomes.
  - Original Message - 
  From: Thiago Tarraf Varella 
  To: OBM Lista 
  Sent: Wednesday, August 10, 2011 1:53 PM
  Subject: RE: [obm-l] 2^70 + 3^70 eh divisivel por 13


  270 + 370 = 24.266 + 31.369 = 24.(26)11 + 3.(33)23 = 16.(64)11 + 3.(27)23 = 
3.(1)11 + 3.(-1)23 = 3.1 + 3.(-1)  = 3 - 3  =  0 (mod 13) 


  Se ele é congruente a 0 em módulo 13, logo ele é divisível por este.
  Abrços :)



--
  From: qed_te...@hotmail.com
  To: obm-l@mat.puc-rio.br
  Subject: [obm-l] 2^70 + 3^70 eh divisivel por 13
  Date: Wed, 10 Aug 2011 16:17:45 +


  Sauda,c~oes, 

  Alguém poderia resolver? 

  Solicitaria a voce uma solução para a questão : 
  demonstre que 270 + 370 é divisível por 13. 


  []'s 
  Luis 



Re: [obm-l] 2^70 + 3^70 eh divisivel por 13

2011-08-30 Por tôpico Carlos Gomes
Eu fiz assim, se a e b são inteiros e n é impar 

a+b | a^n+b^n

Assim,

2^70+3^70 = (2^2)^35+(3^2)^35. Ora como 35 é ímpar segue que 2^2+3^2 |  
(2^2)^35+(3^2)^35  ==  13|2^70+3^70 

Cgomes.
  - Original Message - 
  From: Thiago Tarraf Varella 
  To: OBM Lista 
  Sent: Wednesday, August 10, 2011 1:53 PM
  Subject: RE: [obm-l] 2^70 + 3^70 eh divisivel por 13


  270 + 370 = 24.266 + 31.369 = 24.(26)11 + 3.(33)23 = 16.(64)11 + 3.(27)23 = 
3.(1)11 + 3.(-1)23 = 3.1 + 3.(-1)  = 3 - 3  =  0 (mod 13)


  Se ele é congruente a 0 em módulo 13, logo ele é divisível por este.
  Abrços :)



--
  From: qed_te...@hotmail.com
  To: obm-l@mat.puc-rio.br
  Subject: [obm-l] 2^70 + 3^70 eh divisivel por 13
  Date: Wed, 10 Aug 2011 16:17:45 +


  Sauda,c~oes, 

  Alguém poderia resolver? 

  Solicitaria a voce uma solução para a questão : 
  demonstre que 270 + 370 é divisível por 13. 


  []'s 
  Luis 



[obm-l] Piratas

2010-09-04 Por tôpico Carlos Gomes
Olá Caros amigos...depois de alguns meses afastado da lista estou
retornando.

E para começar um novo probleminha

Vários piratas repartiram 1000 moedas de ouro todas iguais. Após a divisão
um dos piratas ficou com mais da metade das moedas. Durante a primeira
noite, para acalmar os ãnimos, o pirata que tinha mais de metade das moedas
deu a cada um dou outros quanto cada um destros outros já possuiam. Após
essa nova partilha, havia um pirata com mais da metade do total de moedas.
Na segunda noite, o procedimento foi repetido; o pirata que tinha mais da
metade das moedas deu a cada um dou outros piratas tantas moedas quanto cada
um já possuia. Assim noite após noite o precedimento foi sempre repetido.
Depois da décima noite nenhum pirata tinha mais da metade do total de
moedas. Determine o número máximo de piratas presentes no grupo.

Este foi o problema 4 da 26° olimpíada de Matemática Argentina - 2009 ,
primeiro nível.

Se alguém já conhece ou se conseguir a resolução gostaria de vê-la.

Obrigado,

Carlos Gomes


[obm-l] Retorno

2010-09-03 Por tôpico Carlos Gomes
olá Caros amigos...depois de alguns meses afastado da lista estou
retornando.

E para começar um novo probleminha

Vários piratas repartiram 1000 moedas de ouro todas iguais. Após a divisão
um dos piratas ficou com mais da metade das moedas. Durante a primeira
noite, para acalmar os ãnimos, o pirata que tinha mais de metade das moedas
deu a cada um dou outros quanto cada um destros outros já possuiam. Após
essa nova partilha, havia um pirata com mais da metade do total de moedas.
Na segunda noite, o procedimento foi repetido; o pirata que tinha mais da
metade das moedas deu a cada um dou outros piratas tantas moedas quanto cada
um já possuia. Assim noite após noite o precedimento foi sempre repetido.
Depois da décima noite nenhum pirata tinha mais da metade do total de
moedas. Determine o número máximo de piratas presentes no grupo.

Este foi o problema 4 da 26° olimpíada de Matemática Argentina - 2009 ,
primeiro nível.

Se alguém já conhece ou se conseguir a resolução gostaria de vê-la.

Obrigado,

Carlos Gomes


[obm-l] Um criador de ovelhas

2009-12-10 Por tôpico Carlos Gomes
Olá amigos...será que algun de vcs já resolveu esta questão ou pode 
resolverfoi da UFCG 2005 2a fase...

Um criador de ovelhas costuma adotar o seguinte critério para selecionar 
animais para abater: do total de n animais escolhe-se p animais. Se a média dos 
pesos  os p animais for maior que a média dos pesos dos n animais mais 25% 
desta média, então os p animais  estão prontos para o abate. No curral há 15 
ovelhas  cujos pesos em média são dados pela tabela abaixo.



N° de animaisPeso/animalPeso(total - kg)



 514 70

 418 72

 216 32

 322 66

 130 30



As possibilidades existentes para que o critério de abate seja satisfeito são  



a) 24  c) 18 e) 21   



b) 20  d) 17   

 





O gabarito é 21 (alternativa E)...não tô achando...



valew, cgomes


Re: [obm-l] ESTUDO DE MATRIZES

2009-11-24 Por tôpico Carlos Gomes
Robério

Duas matrizes A e B são semelhantes quando existe uma matriz invertível P tal 
que A=P^(-1).B.P. 
Suponha, Seja A uma matriz semelhante matriz nula que  O. Assim existe uma 
matriz invertível P tal que 
A=P^(-1).0.P 
mas 
P^(-1).0.P=0. 
Assim A=0, ou seja a única matriz semelhante a matriz nula é ele própria.
 Analogamente se A é semelhante a  Identidade então existe uma matriz 
invertível P tal que A=P^(-1).I.P=P^P=I. 
Assim a única matriz semelhante a identidade é ela própria.

valew, Cgomes
  - Original Message ---
  From: prof.roberio 
  To: obm-l@mat.puc-rio.br 
  Sent: Tuesday, November 24, 2009 9:30 AM
  Subject: [obm-l] ESTUDO DE MATRIZES



  3) Demonstre que a única matriz semelhante à matriz nula é a própria. Idem 
para a matriz identidade.


  = 
Instru��es para entrar na lista, sair da lista e usar a lista em 
http://www.mat.puc-rio.br/~obmlistas/obm-l.html 
=


--



  No virus found in this incoming message.
  Checked by AVG - www.avg.com 
  Version: 8.5.425 / Virus Database: 270.14.80/2523 - Release Date: 11/24/09 
07:46:00


[obm-l] Irracionais no conjunto de Cantor

2009-11-16 Por tôpico Carlos Gomes
Olá caros amigos

Como posso fazer para saber se um número irracional entre 0 e 1,  pertence ou 
não ao  conjunto de Cantor?

valew, um forte abraço à todos, Cgomes

Re: [obm-l] Algebra Linear II

2009-11-09 Por tôpico Carlos Gomes
lembrando que detM=detM^t  temos:

Os autovalores de A são as raízes do polinômio p(x)=det(A-x.I)

e os de A^t são as raízes do polinômio q(x)=det(A^t-x.I) = det[A^t-x.I^t] = 
det[(A-x.I)^t] = det(A-x.I)=p(x)

assim A e A^t possuem os mesmos autovalores.


valew, cgomes
  - Original Message - 
  From: warley ferreira 
  To: Lista de Discussão 
  Sent: Monday, November 09, 2009 3:34 PM
  Subject: [obm-l] Algebra Linear II


Olá pessoal, td bom?
Queria uma ajuda nesta questão:
Prove que uma matriz A e sua transposta AT possuem os mesmos valores 
próprios.
Desde já agradeço,
Obrigado!
Otávio Souza 


--
  Veja quais são os assuntos do momento no Yahoo! + Buscados: Top 10 - 
Celebridades - Música - Esportes


--



  No virus found in this incoming message.
  Checked by AVG - www.avg.com 
  Version: 8.5.425 / Virus Database: 270.14.56/2491 - Release Date: 11/09/09 
07:39:00


[obm-l] Re: [obm-l] Informação

2009-09-13 Por tôpico Carlos Gomes
Rita...vê em

www.4shared.com

tem milhares!
  - Original Message - 
  From: RitaGomes 
  To: OBM Lista 
  Sent: Sunday, September 13, 2009 10:34 AM
  Subject: [obm-l] Informação


  Caros Colegas,

  Alguem pode me informar algum endereço eletronico em que posso baixar algum 
livro de cálculo bom

  Rita Gomes


--



  No virus found in this incoming message.
  Checked by AVG - www.avg.com 
  Version: 8.5.409 / Virus Database: 270.13.94/2367 - Release Date: 09/13/09 
05:50:00


[obm-l] Re: [obm-l] Re: [obm-l] Re: [obm-l] Triâ ngulo e mediana

2009-07-28 Por tôpico Carlos Gomes

Muito obrigado Ralph...um abração!
Cgomes
- Original Message - 
From: Ralph Teixeira ralp...@gmail.com

To: obm-l@mat.puc-rio.br
Sent: Tuesday, July 28, 2009 12:07 AM
Subject: [obm-l] Re: [obm-l] Re: [obm-l] Triângulo e mediana


Estou com o Luiz. Sejam ABC o triangulo, M o medio de BC, e X o tal
circulo inscrito. Suponha spdg que o ponto de tangencia de X com BC
estah em BM. Sejam P e Q os pontos onde o circulo corta a mediana AM.

Como AP=PQ=MQ=x, temos:

Pot(A,X)=2x^2=Pot(M,X)

Agora olhe para as tangentes saindo de A e de M:
2x^2=(p-a)^2=(p-c-a/2)^2
p-a=p-c-a/2
a=2c

Agora, pela Lei das Medianas:
c^2+b^2-2(3x)^2=a^2/2
Botando a=2c e 2x^2=(p-a)^2=((b-c)/2)^2, vem
c^2+b^2-9(b-c)^2/4=2c^2
Daqui sai b=2.6c ou b=c. A solucao b=c nao presta (triangulo c,c,2c
degenerado). Entao o triangulo tem lados c,2c e 2.6c, ou seja, eh o
triangulo 5,10,13, com alguma semelhanca. Como este aqui tem area
S=raiz(p(p-a)(p-b)(p-c))=raiz(14.9.4.1)=6raiz(14), eu dei sorte e
achei os lados: 5, 10 e 13.

Abraco, Ralph.

P.S.: Se eu nao tivesse dado sorte, usaria uma razao de semelhanca k
para modificar os lados de maneira a chegar aa area pedida.


2009/7/27 luiz silva luizfelipec...@yahoo.com.br:

Ola Carlos,

Não conhecia.

Aparentemente, o que vou descrever gera a uma solução (não fiz as contas) 
:

se usarmos potência, conseguiremos determinar os lados do triângulo em
função de duas variáveis a e b. Após isso, pode-se expressar a mediana em
função de uma destas variáváveis (novamente, através da potência de um
ponto) e, através da fórmula da mediana, podemos encontrar a em função de 
b
ou b em função de a. Assim, teremos os lados expressos através de uma 
única

variável.

Podemos agora usar a fórmula da área com o perímetro, para achar o valor 
da

variável que aparece nas expressões representando os lados e, assim,
determinar os lados do triângulo.

Particularmente, achei essa possível solução muito braçal..por isso não
fiz as contas...sendo assim, com certeza deve haver uma soluão mais
elegante.

Abs
Felipe

--- Em dom, 26/7/09, Carlos Gomes cgomes...@uol.com.br escreveu:

De: Carlos Gomes cgomes...@uol.com.br
Assunto: [obm-l] Triângulo e mediana
Para: obm-l obm-l@mat.puc-rio.br
Data: Domingo, 26 de Julho de 2009, 23:57

Olá gente...alguém conhece essa?

O Circulo inscrito no triângulo ABC divide mediana traçada de A em três
segmentos de mesma medida. Se a área de ABC é 6.Raiz(14). Calcule as 
medidas

dos lados desse triângulo.

valew, cgomes


Veja quais são os assuntos do momento no Yahoo! + Buscados: Top 10 -
Celebridades - Música - Esportes


=
Instruções para entrar na lista, sair da lista e usar a lista em
http://www.mat.puc-rio.br/~obmlistas/obm-l.html
=






No virus found in this incoming message.
Checked by AVG - www.avg.com
Version: 8.5.392 / Virus Database: 270.13.33/2267 - Release Date: 07/27/09 
17:59:00


=
Instruções para entrar na lista, sair da lista e usar a lista em
http://www.mat.puc-rio.br/~obmlistas/obm-l.html
=


[obm-l] Triângulo e mediana

2009-07-26 Por tôpico Carlos Gomes
Olá gente...alguém conhece essa?

O Circulo inscrito no triângulo ABC divide  mediana traçada de A em três 
segmentos de mesma medida. Se a área de ABC é 6.Raiz(14). Calcule as medidas 
dos lados desse triângulo.

valew, cgomes

[obm-l] soma de quadrados

2009-06-25 Por tôpico Carlos Gomes
Olá pessoal...alguém conhece a solução do problema a seguir?

Mostre que não existem inteiros x, y e z tais que

800.000.007=x^2+y^2+z^2

valew, cgomes

[obm-l] ax^5+by^5

2009-05-18 Por tôpico Carlos Gomes
Olá pessoal alguem conhece a solução da questão abaixo?

sabendo que a,b x e y são números reais tais que

ax+by=2 
a^2.x + b^2.y=20
a^3.x + b^3.y=56
a^4.x + b^4.y=272

determine o valor dea^5.x + b^5.y

obrigado,

cgomes

Re: [obm-l] ax^5+by^5

2009-05-18 Por tôpico Carlos Gomes
Valew, Carlos obrigado! sua saida foi simples e muito eficiente!

um abraço,

cgomes
  - Original Message - 
  From: Carlos Alberto da Silva Victor 
  To: obm-l@mat.puc-rio.br 
  Sent: Monday, May 18, 2009 3:52 PM
  Subject: Re: [obm-l] ax^5+by^5


  Olá  Carlos ,
  Multiplique  a  segunda igualdade por  a  , depois por b e adicione os  
resultados . Colocando a.b  em evidência  convenientemente e usando a primeira 
igualdade , você  encontrará  uma  relação  entre  a+b  e a.b .Faça  a mesma  
coisa  para  a terceira igualdade  e encontrarás uma  outra  relação  entre  
a+b  e a.b .Resolvendo o sistema determine o valor  de  a+b e a.b . Faça  agora 
a mesma  coisa  para  a última  igualdade  e  o valor  da  expressão  pedida  
será  determinada , ok ? 

  Abraços 

  Carlos  Victor


   
  2009/5/18 Carlos Gomes cgomes...@uol.com.br

Olá pessoal alguem conhece a solução da questão abaixo?

sabendo que a,b x e y são números reais tais que

ax+by=2 
a^2.x + b^2.y=20
a^3.x + b^3.y=56
a^4.x + b^4.y=272

determine o valor dea^5.x + b^5.y

obrigado,

cgomes




--


  Internal Virus Database is out-of-date.
  Checked by AVG. 
  Version: 7.5.557 / Virus Database: 270.12.11/2089 - Release Date: 30/4/2009 
17:53


[obm-l] Re: [obm-l] Permutação

2009-03-31 Por tôpico Carlos Gomes



consegui...  são  13...basta perceber que o 102 só pode ocupar as posições 
ak em  que k é divisor de 102.








 a1
a2
a3
a4
a5
a6
...
a17
...
a34
...
a51
...
a101

 102
2
3
4
5
6
 17
 34
 51
 101

 2
102
3
4
5
6
 17
 34
 51
 101

 3
2
102
4
5
6
 17
 34
 51
 101

 3
2
6
4
5
102
 17
 34
 51
 101

 6
2
3
4
5
102
 17
 34
 51
 101

 2
6
3
4
5
102
 17
 34
 51
 101

 3
2
6
4
5
102
 17
 34
 51
 101

 17
2
3
4
5
6
 102
 34
 51
 101

 2
34
3
4
5
6
 17
 102
 51
 101

 34
2
3
4
5
6
 17
 102
 51
 101

 51
2
3
4
5
6
 17
 34
 102
 101

 3
2
51
4
5
6
 17
 34
 102
 101

 17
2
3
4
5
6
 51
 34
 102
 101





cgomes

- Original Message - 
From: Benedito b...@ccet.ufrn.br

To: obm-l@mat.puc-rio.br
Sent: Tuesday, March 31, 2009 10:09 AM
Subject: [obm-l] Permutação


Problema legal:

Seja S = {2, 3, 4,, 101, 102}.
Seja  (a1, a2, ..., a101)  uma permutação qualquer  do conjunto S.
Encontre  quantas são as permutações  de S tais que k divide ak.
Benedito

=
Instruções para entrar na lista, sair da lista e usar a lista em
http://www.mat.puc-rio.br/~obmlistas/obm-l.html
=


--
No virus found in this incoming message.
Checked by AVG.
Version: 7.5.557 / Virus Database: 270.11.34/2032 - Release Date: 31/3/2009 
06:02



=
Instruções para entrar na lista, sair da lista e usar a lista em
http://www.mat.puc-rio.br/~obmlistas/obm-l.html
=


[obm-l] Re: [obm-l] Prova do teorema fundamenta da álgebra por análise complexa

2009-03-26 Por tôpico Carlos Gomes
Existe sim Ana...vc usa o Teorema de Liouville que fala que uma função 
inteira limitada é constante. Veja o livro do Alcides Lins do IMPA na pág 
199...está lá demonstrado o TFA


um abraço, Cgomes


- Original Message - 
From: Ana ana...@yahoo.com

To: obm-l@mat.puc-rio.br
Sent: Thursday, March 26, 2009 9:37 PM
Subject: [obm-l] Prova do teorema fundamenta da álgebra por análise complexa



Oi a todos

Soube que, por análise complexa, há uma prova do teorema fundamental da 
álgebra bem mais curta do que por álgebra. Alguém a conhece? Exige 
conhecimentos profundos?


Obrigada
Ana












=
Instruções para entrar na lista, sair da lista e usar a lista em
http://www.mat.puc-rio.br/~obmlistas/obm-l.html
=


--
No virus found in this incoming message.
Checked by AVG.
Version: 7.5.557 / Virus Database: 270.11.29/2024 - Release Date: 26/3/2009 
07:12



=
Instruções para entrar na lista, sair da lista e usar a lista em
http://www.mat.puc-rio.br/~obmlistas/obm-l.html
=


[obm-l] Re: [obm-l] Re: [obm-l] Ajuda exercício

2009-03-05 Por tôpico Carlos Gomes
Oi Rauryson

se a cada bombeada são retirados 20% então restam 80%, assim apos 10 bombeadas 
0 volume remanescente no tanque será 2 x 0,8^10=0,21m^3

valew, cgomes
  - Original Message - 
  From: Rauryson Alves 
  To: obm-l@mat.puc-rio.br 
  Sent: Wednesday, March 04, 2009 7:49 PM
  Subject: [obm-l] Re: [obm-l] Ajuda exercício


tente usar uma pg de razão 0,8

--- Em qua, 4/3/09, Rodrigo Assis rossoas...@gmail.com escreveu:

  De: Rodrigo Assis rossoas...@gmail.com
  Assunto: [obm-l] Ajuda exercício
  Para: obm-l@mat.puc-rio.br
  Data: Quarta-feira, 4 de Março de 2009, 15:40


  Colegas, eu consegui resolver o problema abaixo, mas gostaria de 
conhecer outros métodos:
  Em um tanque de aco, existem cerca de 2m3 de ar. Uma bomba de vacuo 
é instalada retirando, a cada bombeada, 20% do ar. Após 10 bombeadas, qual será 
o volume de ar contido no tanque?

  Eu fiz da pior maneira possível, jogando os 20% 10 vezes. Tentei usar 
progressao, mas nao consegui.

  grato,


   


--
  Veja quais são os assuntos do momento no Yahoo! + Buscados: Top 10 - 
Celebridades - Música - Esportes


--


  No virus found in this incoming message.
  Checked by AVG. 
  Version: 7.5.557 / Virus Database: 270.11.7/1983 - Release Date: 4/3/2009 
07:41


[obm-l] Re: [obm-l] sites de matemática Análise

2009-03-05 Por tôpico Carlos Gomes
Oi Israel...vê essa aí
http://www.math.unl.edu/~webnotes/contents/Classes.htm

gosto muito!

valew, cgomes
  - Original Message - 
  From: Israel Vallin 
  To: obm-l@mat.puc-rio.br 
  Sent: Thursday, March 05, 2009 11:06 AM
  Subject: [obm-l] sites de matemática Análise


  Alguem tem indicações sobre sites de matematica com exemplos de exercicios de 
analise?


--
  Veja mapas e encontre as melhores rotas para fugir do trânsito com o Live 
Search Maps! Experimente já! 


--


  No virus found in this incoming message.
  Checked by AVG. 
  Version: 7.5.557 / Virus Database: 270.11.8/1985 - Release Date: 5/3/2009 
07:54


Re: [obm-l] ajuda!!!!

2009-02-17 Por tôpico Carlos Gomes

Oi Débora,

Eu sugiro  que pense assim.
Sejam A e B os dois trabalhadores. Vamos calcular as velocidades de traalho 
de cada um deles

Va = S/Ta
Vb=S/Tb

onde S representa a área do muro

Ta e Tb são os tempos em dias que A e B levam para (individualmente) para 
construir o muro de área S.


Quando A e B trabalham juntos a velocidade de produção é  Va+Vb = S/4

como Ta=5dias temos que Va=S/5

assim,

Va+Vb = S/4   == S/5+S/Tb=S/4   == Tb=20dias

espero que eu tenha sido claro,

um abraço Débora!

- Original Message - 
From: Debora Bagatin deborab...@ufpr.br

To: obm-l@mat.puc-rio.br
Sent: Tuesday, February 17, 2009 6:34 PM
Subject: [obm-l] ajuda


DOIS OPERÁRIOS CONSTROEM UM MURO EM QUATRO DIAS. UM DELES, TRABALHANDO
SOZINHO, CONSTRÓI O MESMO MURO EM 5 DIAS. PERGUNTA-SE: EM QUANTOS DIAS O
OUTRO OPERÁRIO, TRABALHANDO SOZINHO, CONSEGUIRIA EXECUTAR A MESMA TAREFA?


Este exercício se encontra no conteúdo de regra de tres simples de um
livro do ensino fundamental. A resposta indicada é 20 dias, porem nao
consigo encontrar uma lógica na sua resolução.

Alguem pode me ajudar?

Obrigado

Debora

=
Instruções para entrar na lista, sair da lista e usar a lista em
http://www.mat.puc-rio.br/~obmlistas/obm-l.html
=


--
No virus found in this incoming message.
Checked by AVG.
Version: 7.5.552 / Virus Database: 270.10.25/1957 - Release Date: 17/2/2009 
07:07



=
Instruções para entrar na lista, sair da lista e usar a lista em
http://www.mat.puc-rio.br/~obmlistas/obm-l.html
=


[obm-l] áreas iguais

2008-10-16 Por tôpico Carlos Gomes
Olá amigos...

será que alguém conhece a saída para o problema

Admitamos que exista uma classe se subconjuntos do plano R^2, chamados as 
figuras geométricas, ou seimplesmente, as figuras, com a seguinte 
propriedade:

Dada uma figura F, cada reta ax+by+c=0 reparte F em duas regiões F1 e F2, tais 
que a diferença área(F1)-área(F2) depende continuamente dos parâmetros a, b, e 
c da reta. Prova a partir daí que duas figuras quaisquer F,G contidas no R^2 
existe uma reta do plano que determina decomposições F=F1UF2 e G=G1UG2 com 
área(F!)=área(F2) e área(G1)=área(G2).


obs. É claro que vamos montar uma função apropriada e usar o TVI, mas não estou 
conseguindo montar a função!

se alguem conhecer , gostaria da ajuda!

um abraço à todos,

obrigado, Cgomes

[obm-l] Teste

2008-07-30 Por tôpico Carlos Gomes

- Original Message - 
From: Carlos Gomes 
To: obm-l@mat.puc-rio.br 
Sent: Wednesday, July 30, 2008 6:01 PM
Subject: Teste


Desculpem  o off-topic as estou testando o meu novo email que é [EMAIL 
PROTECTED]

Valew...um abraço para todos.

[obm-l] Teste

2008-07-30 Por tôpico Carlos Gomes
Desculpem  o off-topic as estou testando o meu novo email que é [EMAIL 
PROTECTED]

Valew...um abraço para todos.

[obm-l] teste

2008-07-21 Por tôpico Carlos Gomes
teste

[obm-l] Re: [obm-l] Livro de Cálculo

2008-07-09 Por tôpico Carlos Gomes
Os vols 1,2,3 e 4 do Guidorrzzi  LTC são ótimos!
  - Original Message - 
  From: [EMAIL PROTECTED] 
  To: obm-l@mat.puc-rio.br 
  Sent: Wednesday, July 09, 2008 9:44 AM
  Subject: [obm-l] Livro de Cálculo


  Como vão?


  Eu tenho os dois volumes do Cálculo de Stewart, mas eu gostaria de um livro 
que aborda a matemática de forma mais pura, alguém teria uma sugestão?


  Grato,


  Ass.: Alamir Rodrigues Rangel Jr.

  = 
Instru絥s para entrar na lista, sair da lista e usar a lista em 
http://www.mat.puc-rio.br/~obmlistas/obm-l.html 
= 

[obm-l] Re: [obm-l] analise comb.(difícil)

2008-06-05 Por tôpico Carlos Gomes
Gustavo,  vamos arrumar os números de 1 a 20 da seguinte forma:

12345
6789   10
111213  14  15
161718  19  20


Note que numa mesma coluna estão os números que deixam o mesmo resto quando 
divididos por 5.Como queremos escoher, dentre  os 20 números, 5 números 
distintos onde pelo menos 2 deixam o mesmo resto quando divididos por 5 , temos 
:

N° total de modos de escolher 5 números distintos de 1 a 20   === C(20,5)=15504

N° total de modos de escolher 5 números com restos diferentes (quando divididos 
por 5) == Para isto você deve pegar um em cada coluna o que pode ser feito de 
4.4.4.4.4=1024 modos

assim o nímero de possibilidade procuradas é 15504-1024=14480.



Gustavo,

Note que esxistem três tipos de números

I. Os que virados de cabeça para baixo não representam nenhum número (por 
exemplo, 45.189)

II. Os que virados de cabeça para baixo representam o mesmo número (por 
exemplo, 86198)

III. Os que virados de cabeça para baixo representam números diferentes ( por 
exemplo, 66810)

Note que  os cartões capazes de economizar são os terceiro tipo pois esses 
podem representar dois números. Vamos contar quantos são os números que podem 
ser representados por um mesmo cartão. Para isso perceba que os possíveis 
algarismos de serem virados e ainda assim representarem o mesmo número no 
cartão são 0,1,6,8 e 9. Assim tems 5.5.5.5.5=3125 possibilidades de fabricar um 
número de 5 algarismos com os digitos 0,1,6,8 e 9. Mas destes 3125 números 
quantos são do tipo II?

Note que para ser do tipo II , as casas das extremidades devem der 0 e 0  ou  1 
e 1   ou  6 e 9   ou 8  e  8  ou 9 e 6, tendo assim 5 possibilidades, para a 
segunda e quarta casas as mesmas 5 possibilidade e finalmente para a casa 
central 0 , 1 ou 8  que são 3 possibilidades, havendo portanto 5.5.3=75 cartões 
que de cabeça para baixo podem representar o mesmo número. Assim os números do 
tipo III são 3125-75=3050. Como cada cartão pode representar dois deses números 
segue que nesta modalidade o número de cartões necessãrios é 3050/2=1525.

Assim o número mínimo de cartões para representat os 10 números de cinco 
algarismos é 100.000-1525=98475


Valeu,

Cgomes

- Original Message - 
  From: Gustavo Duarte 
  To: Olimpíada 
  Sent: Thursday, June 05, 2008 8:28 PM
  Subject: [obm-l] analise comb.(difícil)


  Qualquer ajuda é bem vinda, tive dificuldade nas duas !!desde já agradeço.

  1)Escolhemos 5 números ,sem repetição, dentre os inteiros de 1 a 20.Calcule 
quantas escolhas distintas podemser feitas, sabendo que ao menos dois dos 5 
números selecioneodos devem deixar o mesmo resto quando dividido por 5.

  2) Escrevem-se números de 5 algarismo (inclusive começando por Zero)) em 
cartões. Como 0, 1 e 8 não se alteram de cabeça para baixo e como o 6 de cabeça 
para baixo vira 9, um só cartão pode representar dois números ( por exemplo 
06198 e 86190). Qual o número mínimo de cartões para representar todos os 
números de 5 algarismo ?



  __ Informação do NOD32 IMON 1.1189 (20050808) __

  Esta mensagem foi verificada pelo NOD32 sistema antivírus
  http://www.eset.com.br


Re: [obm-l] Questao de um concurso

2008-06-04 Por tôpico Carlos Gomes
Oi José...eu penso o seguinte:

N° de amarelas  = 0,7.n

Mas N° de amarelas = 24 + 6/9.(n-3)

assim.,

24 + 6/9.(n-30) = 0,7.n

216+6n-180 =6,3.n

36  = 0,3.n

n=120


assim o n° máximo é 120

Acho que é isso!

Valeu,  Cgomes
  - Original Message - 
  From: Jose Aurimenes 
  To: obm-l@mat.puc-rio.br 
  Sent: Wednesday, June 04, 2008 8:47 AM
  Subject: [obm-l] Questao de um concurso





 01. De um conjunto de  n   balas coloridas, das quais algumas sao verdes e 
as demais amarelas, observou-se que 24 das 30 primeiras eram amarelas. Em  
seguida,observou-se que 6 de cada 9 contadas eram amarelas. Se no total 70% ou 
mais das balas contadas eram amarelas, o valor maximo de n eh:

  a) 30
  b) 35
  c) 40
  d) 84
 e) 120

 Pessoal gostaria da opiniao de voces, se possivel com a solucao, pois o 
gabarito eh letra A e nao concordo, desde jah agradeco.

Aurimenes

[obm-l] Números algébricos

2008-02-18 Por tôpico Carlos Gomes
Olá amigos...


Quais são os valores  naturais de x para os quais  senx°  é um número algébrico?

Cgomes


[obm-l] Re: [obm-l] função contínua

2008-02-12 Por tôpico Carlos Gomes
Valew  pela  força  Artur!  Por coincidência acabei de encontrar num outro 
livro (PROBLEM SOLVING THROUGH PROBLEMS do  Loren Larson) um problema 
relacionado, na verdade uma generalização que me permitiu resolver o 
problema original. Por coincidência ia por aqui na lista agora.


A generalização é a seguinte:

Seja f:[0,1] --- R, contínua e derivável em (0,1) com f(1)=1 e f(0)=0, 
então para cada inteiro positivo n existem pontos distintos x_1, x_2, ..., 
x_n em (0,1)  tais que


1/f '(x_1)  +  1/f '(x_2) +  +1/f '(x_n) = n

é muito legal.está resolvido na pág 223 do livro que citei acima

Valew, Artur, obrigado pelo interesse.

- Original Message - 
From: Artur Costa Steiner [EMAIL PROTECTED]

To: obm-l@mat.puc-rio.br
Sent: Tuesday, February 12, 2008 11:42 PM
Subject: Re: [obm-l] função contínua


Como f é continua, existe c em (a , b) tal que f(c) =
(a+b)/2.

Aplicando o TVM a [a , c], obtemos x1 em (a , c) tal
que f'(x1) = (f(c) -f(a))/(c -a) =(b - a)/(2(c - a).
Aplicando o TVM agora a [c , b], obtemos x2 em (c , b)
tal que f'(x2) = (f(b) -f(c))/(b - c) =(b - a)/(2(b -
c).

Temos, entao, que a  x1  x2  b e que

1/f'(x1) +
1/f'(x2) = (2(c - a))/(b - a) + (2(b - c))/(b - a =
(2(b - a))/(b - a) = 2 , provando a afirmacao.

Artur

Ps. O merito desta prova nao e meu, um amigo sugeriu o
ponto chave c e eu so dei os arremates finais com o
TVM.


From: Carlos Gomes
To: obm-l@mat.puc-rio.br
Sent: Saturday, February 09, 2008 7:45 AM
Subject: função contínua


Olá amigos...será que alguém pode me ajudar com
essa?

Seja f uma função contínua em [a,b] e
diferenciável em (a,b) tal que f(a)=a e f(b)=b.
Mostre que existem x_1 e x_2 tais que a x_1  x_2 
b tais que 1/f ' (x_1)  +  1/f ' (x_2) = 2.


Valew, Cgomes




 

Be a better friend, newshound, and
know-it-all with Yahoo! Mobile.  Try it now. 
http://mobile.yahoo.com/;_ylt=Ahu06i62sR8HDtDypao8Wcj9tAcJ


=
Instruções para entrar na lista, sair da lista e usar a lista em
http://www.mat.puc-rio.br/~obmlistas/obm-l.html
=

=
Instruções para entrar na lista, sair da lista e usar a lista em
http://www.mat.puc-rio.br/~obmlistas/obm-l.html
=


[obm-l] Re: [obm-l] função contínua

2008-02-11 Por tôpico Carlos Gomes
Artur, note que f(a)=a e f(b)=b , como ab segue que f(a)f(b). Assim f não 
pode ser estritamente decrescente, não acha? Quanto ao enunciado é esse mesmo. 
Esta questão está na pág 107 ( questão 19) do livro ADVANCED CALCULUS 
Autor: Angus E. Taylor

valew

Cgomes
  - Original Message - 
  From: Artur Costa Steiner 
  To: obm-l@mat.puc-rio.br 
  Sent: Monday, February 11, 2008 2:50 PM
  Subject: RES: [obm-l] função contínua


  Da forma como está colocada, a afirmação não é verdadeira. 

  Se f for estritamente decrescente, então f' eh sempre negativa em (a, b) e 
não ha como a sua expressão dar 2, pois é sempre negativa. 

  Nao estah faltando aguma hipotese?
  Artur
-Mensagem original-
De: [EMAIL PROTECTED] [mailto:[EMAIL PROTECTED] nome de Carlos Gomes
Enviada em: domingo, 10 de fevereiro de 2008 09:42
Para: obm-l@mat.puc-rio.br
Assunto: [obm-l] função contínua



- Original Message - 
From: Carlos Gomes 
To: obm-l@mat.puc-rio.br 
Sent: Saturday, February 09, 2008 7:45 AM
Subject: função contínua


Olá amigos...será que alguém pode me ajudar com essa?

Seja f uma função contínua em [a,b] e diferenciável em (a,b) tal que f(a)=a 
e f(b)=b. Mostre que existem x_1 e x_2 tais que a x_1  x_2  b tais que 1/f ' 
(x_1)  +  1/f ' (x_2) = 2.


Valew, Cgomes


  __ Informação do NOD32 IMON 1.1189 (20050808) __

  Esta mensagem foi verificada pelo NOD32 sistema antivírus
  http://www.eset.com.br


[obm-l] função contínua

2008-02-11 Por tôpico Carlos Gomes
Olá amigos...será que alguém pode me ajudar com essa?

Seja f uma função contínua em [a,b] e diferenciável em (a,b) tal que f(a)=a e 
f(b)=b. Mostre que existem x_1 e x_2 tais que a x_1  x_2  b tais que 1/f ' 
(x_1)  +  1/f ' (x_2) = 2.


Valew, Cgomes

[obm-l] função contínua

2008-02-11 Por tôpico Carlos Gomes

- Original Message - 
From: Carlos Gomes 
To: obm-l@mat.puc-rio.br 
Sent: Saturday, February 09, 2008 7:45 AM
Subject: função contínua


Olá amigos...será que alguém pode me ajudar com essa?

Seja f uma função contínua em [a,b] e diferenciável em (a,b) tal que f(a)=a e 
f(b)=b. Mostre que existem x_1 e x_2 tais que a x_1  x_2  b tais que 1/f ' 
(x_1)  +  1/f ' (x_2) = 2.


Valew, Cgomes

[obm-l] Ortocentro....

2007-10-23 Por tôpico Carlos Gomes
O problema abaixo o Ralph (muito gentilmente) resolveu para mim mas caímos numa 
equação do 3° grau que tivemos que usar o cmputador para obter uma raiz 
aproximada...será que alguem aqui na lista conhece outra solução?

Dadas as distâncias a, b e c do ortocentro de um triângulo acutângulo a cada um 
dos seus vértices, determinar a medida da área desse triângulo.

Valew, um abraço a todos

cgomes

[obm-l] Re: [obm-l] Combinatória - album de figurinhas...

2007-10-07 Por tôpico Carlos Gomes

Acho que a pergunta saria o tempo médio para completar o albumnão?
tenho o pequeno artigo  de minha autoria sobre isto vou mandar no seu 
email...

Cgomes


- Original Message - 
From: Carlos Nehab [EMAIL PROTECTED]

To: obm-l@mat.puc-rio.br
Sent: Friday, October 05, 2007 9:40 PM
Subject: [obm-l] Combinatória - album de figurinhas...


Oi, gente,

Considere uma revista de figurinhas  com N figurinhas distintas.  Qual
o número médio de figurinhas que se deve comprar para completar o
álbum?   Imaginem que as figurinhas são compradas unitariamente (uma a uma).

Nehab

PS: Quem sabe o Palmerim bota na coleção dele...

=
Instruções para entrar na lista, sair da lista e usar a lista em
http://www.mat.puc-rio.br/~nicolau/olimp/obm-l.html
=


__ Informação do NOD32 IMON 1.1189 (20050808) __

Esta mensagem foi verificada pelo NOD32 sistema antivírus
http://www.eset.com.br


=
Instruções para entrar na lista, sair da lista e usar a lista em
http://www.mat.puc-rio.br/~nicolau/olimp/obm-l.html
=


[obm-l] Re: [obm-l] transformação

2007-10-03 Por tôpico Carlos Gomes
Não Marcus...está correto pois

cos(2a)=1-2sen^2(a)

fazendo a=2x  ,temos:

cos(2.(2x))=1-2.sen^2(2x)  == cos(4x)=1-2.sen^2(2x)  == sen^(2x) = 
(1-cos(4x))/2

valew,

cgomes
  - Original Message - 
  From: Marcus 
  To: obm-l@mat.puc-rio.br 
  Sent: Tuesday, October 02, 2007 1:53 PM
  Subject: [obm-l] transformação


  Algum pode me dizer se a transformação que to fazendo tem algum erro:

  Sen^2(2x)= (1- cos4x)/2

   

  Marcus Aurélio

   



  __ Informação do NOD32 IMON 1.1189 (20050808) __

  Esta mensagem foi verificada pelo NOD32 sistema antivírus
  http://www.eset.com.br


Re: [obm-l] TRIANGULO ABC

2007-09-28 Por tôpico Carlos Gomes
Mais um...

Como A, B e C são as medidas dos lados de um triângulo temos que:

A+B+C=180°  ==   A+[B+C]=180° ou seja as medidas A e [B+C] são suplementares, 
logo temos que tgA = -tg[B+C] (lembre que quando dois ângulos somam 180° as 
suas tangente têm o mesmo módulo e sinais contrários)

por outro lado, tg[B+C] = (tgB+tgC)/(1-tgB.tgC)


como 2.tgA=tgB+tgC  segue que 

 tgA = -tg[B+C]   ==   tgA = - (tgB+tgC)/(1-tgB.tgC)  ==  tgA = - 2tgA / 
(1-tgB.tgC) 


como 0A90° (de acordo com o enunciado)  temos que neste intervalo tgA é 
sempre diferente de zero e portanto pode ser cancelada em   tgA = - 2tgA / 
(1-tgB.tgC)  o que implica que 

1 = - 2/(1-tgB.tgC)  ==  (1-tgB.tgC) = - 2   =  tgB.tgC=3

alternativa A


valew,

Cgomes


  - Original Message - 
  From: arkon 
  To: obm-l 
  Sent: Friday, September 28, 2007 8:56 AM
  Subject: [obm-l] TRIANGULO ABC


  ALGUÉM PODE, POR FAVOR, RESOLVER ESTA:

   

  (UFPB-77) Num triângulo ABC cujos ângulos são A, B e C. Supõe-se que

  2tg A = tg B + tg C e 0  A  pi/2. Neste triângulo vale a relação:

   

  a) tg B.tg C = 3.   b) cos (B - C) = 2sec A.   c) cos (B + C) = 2cos A.   

  d) tg B.tg C = rq3.e) nenhuma das respostas.

   

  DESDE JÁ MUITO OBRIGADO


[obm-l] Re: [obm-l] EXPRESSÃO

2007-09-28 Por tôpico Carlos Gomes
Vamos la´(Arkon vê se põe parênteses...as vezes fica meio dificil de saber 
exatamente o que quer...valew?)

[secx+1]/[secx-1] = [1/cosx  +  1]/[ 1/cosx - 1]  =  [(1+cosx)/cosx] / 
[(1-cosx)/cosx]  =  [1+cosx] / [1-cosx]

até aí tudo bem neh

agora temos de dar um jeito de fazer com que isto fique igual a alguma das 
alternativas...aí é que vem o truque:


Você deve multiplicar o numerador e o denominador da fração  [1+cosx] / 
[1-cosx]  por [1+cosx] (é como se você estivesse racionalizando  
denominadores...)

veja:

 [1+cosx] / [1-cosx] =  [1+cosx] .[1+cosx] / [1-cosx].[1+cosx]  =  [1+cosx] ^2 
/ 1- sen^2 x = [(1+cosx)/senx]^2 = [1/senx  +  cosx/senx]^2  =  [cossecx  +  
cotgx]^2

que é justamente a alternativa C

Valew, Cgomes
  - Original Message - 
  From: arkon 
  To: obm-l 
  Sent: Friday, September 28, 2007 8:53 AM
  Subject: [obm-l] EXPRESSÃO


  Alguém pode, por favor, resolver esta:

   

  (UFPB-78) A expressão sec x + 1/sec x - 1 é idêntica a:

   

  a) (cos x + cotg x)2. b) (sen x + cotg x)2.   c) (cossec x + cotg x)2.

  d) (sec x + tg x)2.e) (tg x + cotg x)2.

   

  DESDE JÁ MUITO OBRIGADO


[obm-l] Re: [obm-l] EXPRESSÃO

2007-09-28 Por tôpico Carlos Gomes
Vamos la´(Arkon vê se põe parênteses...as vezes fica meio dificil de saber 
exatamente o que quer...valew?)

[secx+1]/[secx-1] = [1/cosx  +  1]/[ 1/cosx - 1]  =  [(1+cosx)/cosx] / 
[(1-cosx)/cosx]  =  [1+cosx] / [1-cosx]

até aí tudo bem neh

agora temos de dar um jeito de fazer com que isto fique igual a alguma das 
alternativas...aí é que vem o truque:


Você deve multiplicar o numerador e o denominador da fração  [1+cosx] / 
[1-cosx]  por [1+cosx] (é como se você estivesse racionalizando  
denominadores...)

veja:

 [1+cosx] / [1-cosx] =  [1+cosx] .[1+cosx] / [1-cosx].[1+cosx]  =  [1+cosx] ^2 
/ 1- sen^2 x = [(1+cosx)/senx]^2 = [1/senx  +  cosx/senx]^2  =  [cossecx  +  
cotgx]^2

que é justamente a alternativa C

Valew, Cgomes




  - Original Message - 
  From: arkon 
  To: obm-l 
  Sent: Friday, September 28, 2007 8:53 AM
  Subject: [obm-l] EXPRESSÃO


  Alguém pode, por favor, resolver esta:

   

  (UFPB-78) A expressão sec x + 1/sec x - 1 é idêntica a:

   

  a) (cos x + cotg x)2. b) (sen x + cotg x)2.   c) (cossec x + cotg x)2.

  d) (sec x + tg x)2.e) (tg x + cotg x)2.

   

  DESDE JÁ MUITO OBRIGADO


[obm-l] Re: [obm-l] SEGURANÇAS

2007-09-28 Por tôpico Carlos Gomes
Este é legal

No item (a) basta notar que o político ficaria no baricentro de um triângulo 
equilátero de lado 2m. Lembrando que o segmento que vai de um dos vértices de 
um triângulo até o baricentro é 2/3 da mediana , que num triângulo equilátero o 
comprimento da mediana coincide com o da altura e que a medida da altura de um 
triângulo equilátero de lado x é x.srqt(3)/2 segue que que neste caso a 
distância do político até cada um dos seus seguranças era de 
(2/3).2.srqt(3)/2=2.srqt(3)/3 m.


Agora o (b)

Talvez seja melhor usar um sistema de coordedadas veja:

Imagine um quadrado com um dos vértices na origem O= (0,0). Agora suponha que o 
político esteja no ponto P = (a,b). Como a distância do político a um dos seus 
seguranças é 1, sem perda de generadidade podemos supor que seja o segurença 
que está no ponto O=(0,0). Assim temos que a^2+b^2=1 (pitátoras ou pela fórmula 
da distância entre dois pontos).

Suponhamos agora dois outros seguranças, um no vértice Q=(c,0) e outro no 
vértice R=(c,c) (faça uma figura para acompanhar). Vamos agora imaginar que a 
distância entre o político P=(a,b) e o segurança em Q=(c,0) seja igual a 4  e 
portando a distância do político P=(a,b) ao segurança R=(c,c) igual a 5. Isto 
implica nas seguintes igualdades:

d(P,Q)=4  ==  (c-a)^2 + b^2 = 16

d(P,R)=5  ==  (c-a)^2 + (c-b)^2 = 25

Obs.
( na verdade a distância do político ao outro segurança do ponto R=(c,c) não 
pode ser  igual a 4, visto que se marcarmos um ponto no interior de um quadrado 
e ligarmos este ponto a cada um dos vértices do quadrado pode-se mostrar que a 
soma dos quadrados das distâncias que ligam vértices opostos é constante. Assim 
os segmentos de tamanho 4 e 5 não podem ligar vértices opostos pois teríamos 
1^2 + 4^2 = 5^2 +d^2  == d  0 o que seria impossível!)


temos então que resolver o seguinte sistema:


a^2+b^2=1

(c-a)^2 + b^2 = 16

 (c-a)^2 + (c-b)^2 = 25

na verdade basta achar o c que é justamente a medida do lado do quadrado

façamos o seguinte: primeiro vamos subtarir a 3a eq da 2a equação:   (c-b)^2 - 
b^2 = 9   == b = [c^2-9]/2c

agora vamos desenvolver um pouco a 2a equação:

(c-a)^2 + b^2 = 16  == c^2 - 2ac + a^2 + b^2 = 16 , mas a^2+b^2=1 == c^2 - 
2ac +1 = 16  ==  c^2 -2ac = 15  == a=[c^2 - 15]/ 2c


agora substituindo  b = [c^2-9]/2c  e a=[c^2 - 15]/ 2c  em a^2+b^2=1 

obtemos:

[(c^2 - 15)/ 2c]^2  + [(c^2-9)/2c]^2 = 1== (c^2-15)^2  +  (c^2-9)^2  = 4c^2

fazendo a troca de variável  c^2 = x temos:

(x-15)^2 + (x-9)^2 = 4x  ==

x^2-30x+225 + x^2-18x+81 - 4x = 0  ==

2x^2 - 52x+306=0  ==


x' = 9   ou x'' =17

mas ocorre que c^2 = x, asim temos que c^2 = 9  == c=3 , pois c0

como  b = [c^2-9]/2c  e a=[c^2 - 15]/ 2c   neste caso teríamos que b = 0 e a = 
- 1 , o que é impossível neste caso pois devemos ter a0

Assim concluímos que 

se x= 17  temos c^2 = 17  o que implicaria c=sqrt(17) e portanto b=4/sqrt(17)  
e a=1/sqrt(17) o que é plenamente possível.


finalmente como c é a medida do lado do quadrado segue que a sua área mede c^2 
que é igual a 17.

valew, 

Cgomes









  - Original Message - 
  From: arkon 
  To: obm-l 
  Sent: Friday, September 28, 2007 2:57 PM
  Subject: [obm-l] SEGURANÇAS


  ALGUÉM PODE RESOLVER, POR FAVOR, ESTE PROBLEMINHA CASCUDO:

   

  Um político contrata quatro seguranças para poder participar de um showmício 
de seu partido. Os seguranças localizam-se nos vértices de um quadrado. Sabe-se 
que três deles estão a 1, 4 e 5 m de seu patrão e sempre mantém esta 
configuração. De posse dessas informações, resolva:

   

  a) Numa emergência em que um dos seguranças fosse atingido por uma bala, os 
outros deveriam constituir um formato de um triângulo eqüilátero de lado 2 cm, 
onde o político localizar-se-ia em seu centro. Calcular a distância, em cm, do 
político aos seguranças nesta ocasião. Despreze a parte fracionária do 
resultado, caso exista.

   

  b) Calcular em m2, a área do quadrado.

DESDE JÁ MUITO OBRIGADO

Re: [obm-l] POQUER

2007-09-26 Por tôpico Carlos Gomes
Vamos lá...(acho que o enunciado deveria ser mais preciso)

Cada naipe está com 8 cartas em jogo ( 7,8,9,10,J, K, Q, A). Supondo que 
queremos que um determinado jogador adquira EXATAMENTE 3 ases temos que ele 
pode receber as suas 5 cartas de C(32,5) modos distintos e rereber 3 ases de 
C(4,3) modos distintos e duas cartas que não são ases de C(28,2) modos 
distintos ( POIS DA 32 CARTAS SÃO 4 AESE E 28 QUE NÃO SÃO ASES). Assim, após 
todas essas suposições a probabilidadse pedida seria igual a 

P=C(4,3).C(28,2) / C(32,5).

Na minha opinão não está claro no enunciado que o jogador estaria 
fixadoisto mudaria completamente a solução do problemaproblemas de 
combinatória  e probabilidade devem ter enunciados bem precisos..pena que nem 
sempre isto ocorra...


valew,
Cgomes
  - Original Message - 
  From: arkon 
  To: obm-l 
  Sent: Wednesday, September 26, 2007 9:13 AM
  Subject: [obm-l] POQUER


  PESSOAL ALGUÉM PODE, POR FAVOR, RESOLVER ESTA

   

  Na disputa de uma jogo de pôquer utilizam-se 32 cartas do baralho, sendo 8 de 
cada naipe (do 7 ao ás). São distribuídas cinco cartas para cada jogador. A 
probabilidade de um jogador receber 3 ases é:

   

  a) C4,3 + C28,2/C32,5.  b) C4,3 x C28,2/C32,5.c) A4,3 + A28,2/A32,5.

  d) A4,3 x A28,2/A32,5.  e) A4,3/C32,3.


  DESDE JÁ MUITO OBRIGADO

Re: [obm-l] COMPUTADOR

2007-09-26 Por tôpico Carlos Gomes

Vamos lá

chamemos de data 0 o dia da compra. Então os pagamentos de x reais seriam 
efetuados nas datas 1 , 2, ,3 ,4 e 5 , ou seja,


data 1  == x

data 2  == x

data 3  == x

data 4  == x

data 5  == x

Carlos quer efetuar o pagamento total na data 3 , pagando y reais.  Como a taxa 
de juros mensais é de 8% ao mês o seu fator de correção é igual a 1,08. Assim 
se levarmos as prestações 1 e 2 para a data 3 elas terão valores 1,08^2.x  e 
1,08.x ( note que a cada mês cada prestação ficaria multiplicada por 1,08, no 
caso da primeira prestação multiplicariamos por 1,08 duas vezes, ou seja, por 
1,08^2 pois ela seria adiada por dois meses). Já as prestações 4 e 5 , na data 
3 valeriam x/1,08 e x/1,08^2 respectivamente. ( Note que ao antecipar uma 
prestação um mês você divide o seu valor por 1,08. No caso da 5a prestação vc 
precisaria dividi-la duas vezes por 1,08, isto é, por 1,08^2). Assim na data 3 
o valor a ser desenbolsado por carlos seria

y= 1,08^2.x  +  1,08.x + x  + x/1,08  +  x/1,08^2

pondo o fator comum x em evidência,

y= x.(1,08^2+  1,08+ 1 + 1/1,08  +  1/1,08^2)

passando o x dividindo,

y/x = 1,08^2+  1,08+ 1 + 1/1,08  +  1/1,08^2

agora note que o segundo membro é uma PG de primeiro termo 1,08 , razão 1/1,08 
e 5 termos.


aplicando a fórmula da soma dos n primeiros termos de um PG, Sn= 
a1.(q^n-1)/(q-1) temos (fazendo com  cuidado as continhas)

y/x= [1,08^5 - 1]/0,08.1,08^2

que é a laternativa A

valew 

cgomes


  - Original Message - 
  From: arkon 
  To: obm-l 
  Sent: Wednesday, September 26, 2007 9:05 AM
  Subject: [obm-l] COMPUTADOR


  Alguém pode resolver, por favor, esta: 

  (UNB) Carlos comprou um computador a prazo, em cinco parcelas iguais e 
sucessivas, cada uma delas de valor x, a serem pagas de 30 em 30 dias, vencendo 
a primeira 30 dias após a compra. No dia subseqüente ao fechamento do negócio, 
Carlos decidiu renegociar a dívida, propondo saldá-la com um único pagamento 
(y) no dia do vencimento da terceira parcela do plano original. Se a taxa de 
juros envolvida nessa negociação for de 8% para cada período de 30 dias, para 
que as duas propostas de pagamento do computador sejam equivalentes, o 
quociente y/x deverá ser igual a 

  a) (1,08)5 - 1/0,08(1,08)2.   b) 8(1,08)2/(1,08)5 - 1.  c) 1 - 
(1,08)-5/0,08(1,08)2.

  d) [(1,08)5 - 1]0,08/(1,08)2.  e) (0,08)3[1 - (1,08)-2]/1,08.

  DESDE JÁ MUITO OBRIGADO


Re: [obm-l] COMPUTADOR

2007-09-26 Por tôpico Carlos Gomes
o primeiro termo da PG é 1,08^2 , errei a digitação mas o resultado esta 
correto!
desculpe-me
Cgomes
  - Original Message - 
  From: Carlos Gomes 
  To: obm-l@mat.puc-rio.br 
  Sent: Wednesday, September 26, 2007 6:17 PM
  Subject: Re: [obm-l] COMPUTADOR



  Vamos lá

  chamemos de data 0 o dia da compra. Então os pagamentos de x reais seriam 
efetuados nas datas 1 , 2, ,3 ,4 e 5 , ou seja,


  data 1  == x

  data 2  == x

  data 3  == x

  data 4  == x

  data 5  == x

  Carlos quer efetuar o pagamento total na data 3 , pagando y reais.  Como a 
taxa de juros mensais é de 8% ao mês o seu fator de correção é igual a 1,08. 
Assim se levarmos as prestações 1 e 2 para a data 3 elas terão valores 1,08^2.x 
 e 1,08.x ( note que a cada mês cada prestação ficaria multiplicada por 1,08, 
no caso da primeira prestação multiplicariamos por 1,08 duas vezes, ou seja, 
por 1,08^2 pois ela seria adiada por dois meses). Já as prestações 4 e 5 , na 
data 3 valeriam x/1,08 e x/1,08^2 respectivamente. ( Note que ao antecipar uma 
prestação um mês você divide o seu valor por 1,08. No caso da 5a prestação vc 
precisaria dividi-la duas vezes por 1,08, isto é, por 1,08^2). Assim na data 3 
o valor a ser desenbolsado por carlos seria

  y= 1,08^2.x  +  1,08.x + x  + x/1,08  +  x/1,08^2

  pondo o fator comum x em evidência,

  y= x.(1,08^2+  1,08+ 1 + 1/1,08  +  1/1,08^2)

  passando o x dividindo,

  y/x = 1,08^2+  1,08+ 1 + 1/1,08  +  1/1,08^2

  agora note que o segundo membro é uma PG de primeiro termo 1,08 , razão 
1/1,08 e 5 termos.


  aplicando a fórmula da soma dos n primeiros termos de um PG, Sn= 
a1.(q^n-1)/(q-1) temos (fazendo com  cuidado as continhas)

  y/x= [1,08^5 - 1]/0,08.1,08^2

  que é a laternativa A

  valew 

  cgomes


- Original Message - 
From: arkon 
To: obm-l 
Sent: Wednesday, September 26, 2007 9:05 AM
Subject: [obm-l] COMPUTADOR


Alguém pode resolver, por favor, esta: 

(UNB) Carlos comprou um computador a prazo, em cinco parcelas iguais e 
sucessivas, cada uma delas de valor x, a serem pagas de 30 em 30 dias, vencendo 
a primeira 30 dias após a compra. No dia subseqüente ao fechamento do negócio, 
Carlos decidiu renegociar a dívida, propondo saldá-la com um único pagamento 
(y) no dia do vencimento da terceira parcela do plano original. Se a taxa de 
juros envolvida nessa negociação for de 8% para cada período de 30 dias, para 
que as duas propostas de pagamento do computador sejam equivalentes, o 
quociente y/x deverá ser igual a 

a) (1,08)5 - 1/0,08(1,08)2.   b) 8(1,08)2/(1,08)5 - 1.  c) 1 - 
(1,08)-5/0,08(1,08)2.

d) [(1,08)5 - 1]0,08/(1,08)2.  e) (0,08)3[1 - (1,08)-2]/1,08.

DESDE JÁ MUITO OBRIGADO



  __ Informação do NOD32 IMON 1.1189 (20050808) __

  Esta mensagem foi verificada pelo NOD32 sistema antivírus
  http://www.eset.com.br


  __ Informação do NOD32 IMON 1.1189 (20050808) __

  Esta mensagem foi verificada pelo NOD32 sistema antivírus
  http://www.eset.com.br



--


  No virus found in this incoming message.
  Checked by AVG Free Edition. 
  Version: 7.5.488 / Virus Database: 269.13.32/1032 - Release Date: 26/9/2007 
20:20


  __ Informagco do NOD32 IMON 1.1189 (20050808) __

  Esta mensagem foi verificada pelo NOD32 sistema antivmrus
  http://www.eset.com.br



Re: [obm-l] DETERMINANTE

2007-09-25 Por tôpico Carlos Gomes
Basta notar que para X=A a segunda e a terceira coluna ficam proporcionais  ( a 
segunda coluna será igual a A vezes a terceira coluna) e portanto o 
determinante será nulo, visto se um matriz apresenta duas filas paralelas 
paralelas proporcinais o seu determinande é nulo!

Cgomes
  - Original Message - 
  From: arkon 
  To: obm-l 
  Sent: Tuesday, September 25, 2007 9:00 AM
  Subject: [obm-l] DETERMINANTE


  Pessoal alguém pode, por favor, resolver esta

   

  (UFPB-77) O determinante   | X  A  1  |será nulo para:

  | B  X  1  |

  | 1  A   1  |

   

  a) A = B.b) X = B.c) X = A. d) X = -1. e) Nenhuma das 
respostas.

   

  DESDE JÁ MUITO OBRIGADO


Re: [obm-l] MATRIZ.1

2007-09-25 Por tôpico Carlos Gomes
Basta lembrar que det(A.At)=det(A).det(At) =det(A).det(A) = (detA)^2 , pois 
det(At)=det(A)

Mas de acordo com a matriz dada temos que det(A)=(cosx)^2+(senx)^2 = 1

segue então que det(A.At)=(detA)^2=1^2=1

cgomes
  - Original Message - 
  From: arkon 
  To: obm-l 
  Sent: Tuesday, September 25, 2007 8:58 AM
  Subject: [obm-l] MATRIZ.1


  Pessoal alguém pode, por favor, resolver esta

   

  (UFPB-77) Dada a matriz   A = |   cos x  sen x | , o det (AAt) é 
igual a: 

  | - sen x  cos x |

   

  a) cos2 x - sen2 x.   b) - 1.c) sen2 x - cos2 x.d) 1.   e) 
Nenhuma das respostas.

   

  DESDE JÁ MUITO OBRIGADO


[obm-l] Re: [obm-l] Re:[obm-l] DÍGITOS

2007-09-24 Por tôpico Carlos Gomes
Oi Arkon  td bem...

eh o seguinte existem C(6,4)=15 subconjuntos de 4 elementos que podemos formar 
com os elementos do conjunto {2,3,4,6,7 ,8}. Note que para cada um destes 
subconjuntos com quatro elementos podemos formar um único número cm 4 
algariamos distintos e em ordem decrescente,por exemplo se escolhermos o 
subconjunto {2,3,4,6} poderemos formar 4!=24 números de 4 algarismos distintos 
com os elementos deste conjuno, entretanto desses 24 apenas 1, o número 6432 
apresentará os seus algarismos em ordem descrescente. Assim a quantidade de 
números com 4 algarismos distintos que podemos formar com os digitos 2,3,4,6,7 
e 8 de modo que os algarismos do número formado estejam em ordem decrescente é 
igual a quantidade de subconjuntos de 4 elementos que podemos formar a partir 
do conjunto {2,3,4,6,7 ,8}. Como existem C(6,4)=15 subconjutos segue que 
existirão 15 números respeitando as exigências do enunciado

valew,

Cgomes
  - Original Message - 
  From: arkon 
  To: obm-l 
  Sent: Monday, September 24, 2007 9:50 AM
  Subject: [obm-l] Re:[obm-l] DÍGITOS


  ALGUÉM CONSEGUIU RESOLVER ESTA?  
  Olá pessoal, alguém pode, por favor, resolver esta:

   

  (ESAF) Com os dígitos 2, 3, 4, 6, 7 e 8, quantos números de quatro 
algarismos, que estejam em ordem decrescente, podemos formar?

  a) 3.   b) 120.c) 15. d) 24.  e) 
360..

   

  DESDE JÁ MUITO OBRIGADO

   


Re: Re:[obm-l] E.NAVAL-84

2007-09-24 Por tôpico Carlos Gomes
Vamos mais uma Arkon essa me parece uma pegadinha barata ...quando ele 
diz  4 algarismos significativos distintos me parece que ele quer que o zero 
não seja incluído (acho que seria muito mais elegante da parte do examinador 
ele falar isto abertamente e não ficar insunuandomas tudo bem..)

Note que para a soma dos 4 algarismos ser par eh preciso que

i.  os 4 algarismos sejam pares ou

ii.  sejam 2 algarismos pares e 2 ímpares   ou

iii. os 4 algarismos sejam ímpares

vamos contar separadamente quantos números existem em cada uma destas classes e 
dapois adicionar os resultados


i. existem 5 algarismos pares (2,4,6,8)  para ecolhermos o primeiro temos 4 
possibilidades para o segundo 3 possibilidades ( não podemos escolher novamente 
o número já escolhido para a primeira posição), para o terceiro 2 e finalmente 
para o quarto 1  Assim pelo príncípio multiplicativo segue que existem 
4.3.2.1=24 números nesta classe

ii.Agora queremos 2 pares e dois ímpares, para escolhermos dois pares e dois 
ímpares ( os pares estão ente 2,4,6 ou 8 e os os ímperes entre 1,3,5,7,ou9)  
temos C(4,2).C(5,2)=60 modos distintos. Uma vez escollhidos podemos arrumá-los 
de 4!=24 modos distintos. Assim o total de possibilidades é 60.24=1440 números 
nesta classe

iii.Agora queremos 4 algarismos ímpares, para escolhermos o primeiro temos 5 
possibilidades, o segundo 4 possibilidades, o terceiro 3 e o quarto pode ser 
escolhido de 2 maneiras distintas, assim pelo princípio fundamental da contagem 
temos 5.4.3.2=120 números nesta classe

Assim o total de números inteiros com 4 algarismos significativos distintos é 
24 + 1440 + 120 = 1584.



valew,

Cgomes





- Original Message - 
  From: arkon 
  To: obm-l 
  Sent: Monday, September 24, 2007 10:47 AM
  Subject: Re:[obm-l] E.NAVAL-84


  ALGUÉM CONSEGUIU RESOLVER ESTA?
  Olá, pessoal.

   

  Mais uma da Escola Naval.

   

  Desde já agradeço.

   

  Abraços.

   

  (EN-84) Considere todos os números inteiros com 4 algarismos significativos 
distintos. 

  Quantos, destes números, têm a soma de seus algarismos par?

   

  a) 384.b) 1104.c) 1584.   d) 5904.  e) 3024.

   


[obm-l] é simples

2007-09-02 Por tôpico Carlos Gomes
Olá pessoal...

estou com um probleminha e queria a opiião de vcs...eh uma questão do 
vestibular ITA-73



Durante o eclipse total do sol de 07 de março de 1970

a largura da faixa da escuridão total foi de 100 km. Em

cada ponto do eixo central desta faixa, a duração do

período de escuridão total foi de 3 minutos. Qual foi a

duração deste período num ponto situado a 10 km do

limite da faixa de escuridão total?



a) 1 min. 36 seg. b) 1 min. 48 seg. c) 1 min. 30 seg.

d) 0 min. 36 seg. e) n.d.a.



A alternativa D parece óbviatô meio desconfiado



alguem conhece essa questão?



valew,  cgomes








Re: [obm-l] probabilidade

2007-08-25 Por tôpico Carlos Gomes
É simples! quando você trabalha com  uma distribuição contínua o fato da 
probabilidade de um evento ser zero não siguinifica que o evento seja 
impossível. Por exemplo se escolhermos aleatoriamente um número real no 
itervalo [0,1], qual é a probabilidade desse número ser o número 1/2?   ...é 
zero e nem por isso o evento escolher o número 1/2 é impossível. Em 
síntese , no caso contínup é possível que a probabilidade seja zero sem que 
o evento seja impossível!


Valew, Cgomes
- Original Message - 
From: Antonio Giansante [EMAIL PROTECTED]

To: obm-l@mat.puc-rio.br
Sent: Saturday, August 25, 2007 12:53 PM
Subject: [obm-l] probabilidade



Essa é interessante:

Escolha um número real qualquer. Por exemplo, vou
escolher o 3. Mas isso é impossível, pois a
probabilidade de eu escolher o três é 1/oo -- 0.
Assim, eu não posso escolher o três!! Aliás, eu não
posso escolher número algum!!! Alguém pode me ajudar a
explicar isso?


 Flickr agora em português. Você clica, todo mundo vê.
http://www.flickr.com.br/
=
Instruções para entrar na lista, sair da lista e usar a lista em
http://www.mat.puc-rio.br/~nicolau/olimp/obm-l.html
=

=
Instruções para entrar na lista, sair da lista e usar a lista em
http://www.mat.puc-rio.br/~nicolau/olimp/obm-l.html
=


  1   2   >